Docsity
Docsity

Prepare for your exams
Prepare for your exams

Study with the several resources on Docsity


Earn points to download
Earn points to download

Earn points by helping other students or get them with a premium plan


Guidelines and tips
Guidelines and tips

Unit 7 QuestionsUnit 7 Questions, Exams of Nursing

Unit 7 QuestionsUnit 7 Questions

Typology: Exams

2022/2023

Available from 09/04/2023

Matthewnl
Matthewnl 🇺🇸

142 documents

Partial preview of the text

Download Unit 7 QuestionsUnit 7 Questions and more Exams Nursing in PDF only on Docsity! Most patients with DVT may be treated in the outpatient setting. However, there are multiple contraindications to outpatient treatment, most involving increased bleeding risk, including: active peptic ulcer disease, recent surgery, and weight <55 kg for males and <45 kg for females. Lung cancer or any other cancer that does not involve brain metastases is not a contraindication for outpatient treatment. Leavitt, A.D., & Minichiello, T. (2019). Disorders of Hemostasis, Thrombosis, & Antithrombotic Therapy. In M.A. Papadakis & S.J. McPhee (Eds.), Current medical diagnosis and treatment 2019 (pp. 577-588). New York: McGraw- Hill Education. Peripheral artery disease (PAD) causes intermittent claudication, pulses in the lower extremities to be faint or absent, may cause bruits over the larger arteries, and usually results in an ABI of less than 0.9 (normal is 0.9-1.2). PAD can also cause ischemic and arterial ulcers; however, these are generally found in the toes and feet. Large ulcers near the ankles are characteristic of venous ulcers and chronic venous insufficiency. Gasper, W.J., Rapp, J.H., & Johnson, M.D. (2019). Blood Vessel & Lymphatic Disorders. In M.A. Papadakis & S.J. McPhee (Eds.), Current medical diagnosis and treatment 2019 (pp. 483-501). New York: McGraw-Hill Education. 1 Unit 7 Questions 1. Knowing that treatment for deep vein thrombosis (DVT) involves administration of anticoagulants, which of the following patients can be safely treated for DVT in the outpatient setting? A. an 80-year-old woman who weighs 42 kg B. a 22-year-old man who had an appendectomy 2 days ago C. a 32-year-old woman with peptic ulcer disease D. a 55-year-old man with lung cancer in remission 2. The nurse practitioner is examining a 65-year-old man with a history of type 2 diabetes mellitus and a complaint of cramping pain in his calves when walking. The patient reports the pain is alleviated with rest but returns when the patient must walk again. The nurse practitioner expects to find all of the following on exam consistent with the diagnosis of peripheral artery disease, except: A. weak or absent dorsalis pedis pulses B. large ulcerations at the medial ankles C. bruits over the femoral arteries D. an ABI of 0.6 3. A 43-year-old female presents with complaints of weight gain, constipation, memory fog, and fatigue. Her labs reveal a TSH of 6.7 and Free T4 of 5. Your plan for this patient includes: A. Her labs are within normal range, and no treatment is needed. B. Start her on Synthroid at 1.6mcg/kg/day and recheck labs in 4-6 weeks. C. Instruct her to take her Synthroid on a full stomach for best absorption. D. Start her on Synthroid at 0.8mcg/kg/day and recheck labs in 2 weeks. Page 1 of 114 Normal TSH values are 0.4-4 mIU/L and normal FreeT4 is 10-27pmlol/L. This patient has both the symptoms and lab values for hypothyroidism. Synthroid (synthetic levothyroxine) is the first line medication for hypothyroidism and starting dose is 1.6mcg/kg/day based on ideal body weight. TSH and Free T4 should be rechecked every 4-6 weeks until euthyroid and normal lab values should be obtained within 1-2 months of starting therapy. Fitzgerald, P.A. (2019). Endocrine Disorders. In M.A. Papadakis & S.J. Mcphee (Eds.), Current medical diagnosis and treatment 2019 (pp.1134-1137). New York: McGraw-Hill Education. 1 Page 2 of 114 Antimuscarinic agents such as tolterodine or oxybutynin may provide additional benefit in stress incontinence issues. These medications must be used with caution due to the side effects of dry mouth, urine retention, and delirium. A pessary may also be of additional benefit but should be prescribed only by providers who are experienced in the selection, placement, and management of these devices. Harper G, & Johnston C, & Landefeld C (2021). Overview: geriatric disorders. Papadakis M.A., & McPhee S.J., & Rabow M.W.(Eds.), Current Medical Diagnosis & Treatment 2021. McGraw-Hill. https://accessmedicine-mhmedical- com.libproxy.usouthal.edu/content.aspx?bookid=2957&sectionid=249362728 Acute Bacterial Prostatitis presents with fever, irritative voiding symptoms, perineal and/or suprapubic pain, and a positive UA. Chronic bacterial prostatitis does not present with fever or a positive UA. Acute epididymitis will present with painful scrotal swelling due to enlargement of the epididymitis. Prostatodynia is a noninflammatory disorder presenting with a normal UA and no fever (Papadakis & McPhee, 2019, p. 970-973). Papadakis, M.A., McPhee, S.J. (2019). Urologic Disorders. In M.V., Wang, T.J., Walsh, & T.D., Chi (Eds.), Current medical diagnosis & treatment (58th ed., pp. 970 - 973). United States of America: McGraw-Hill Education. Chlamydial infections were higher in 14- to 24-year-old women who reported same-sex behavior when compared with exclusively heterosexual women. Untreated chlamydial infection places a woman’s future fertility at risk due to potential tubal occlusion. Some women who have a chlamydia infection do not have symptoms. Secondary sequelae of chlamydia include intra-abdominal abscesses, chronic pain, and the need for multiple surgeries. Regardless of sexual orientation, the CDC recommends annual Chlamydia trachomatis (and Neisseria gonorrheae) screening from the age of first sexual activity to the age of 25 years for all women. Compared to heterosexual women, lesbians and bisexual women have higher exposure to violence throughout their lifetimes. The lifetime prevalence of sexual assault may be as high as 85%. It is essential to screen all women for IPV but especially those in the LGBTQ community due to these alarming statistics. The primary prevention of cervical cancer is essential. All females between the ages of 12 and 26 years 1 A. Tolterodine 1-2 mg orally 2x daily B. Oxybutynin 2.5 – 5mg orally 2-3x daily C. Refer to OB/Gyn for a pessary fitting D. All of the above 8. A 23-year-old male patient presents to the clinic with complaints of fever, irritative voiding symptoms, and perineal/suprapubic pain for 2 days. On exam, the CBC shows leukocytosis and a left shift. UA is positive for pyuria and bacteriuria. There is no CVA tenderness or painful scrotal enlargement. The NP suspects the patient has which of the following diagnosis? A. Acute Bacterial Prostatitis B. Chronic Bacterial Prostatitis C. Acute Epididymitis D. Prostatodynia 9. A 24-year-old female who identifies as a lesbian, presents for her annual wellness visit. She has multiple female sexual partners. She did not receive the HPV vaccine and has not had a cervical cancer screening since she was 21 years old. Which of the following is not recommended in the treatment plan for this patient? A. HPV vaccination series B. Papanicolaou (PAP) smear with HPV co-testing C. Chlamydia trachomatis and Neisseria gonorrhea testing D. Screen for Intimate Partner Violence (IPV) Page 5 of 114 Abstinence, a monogamous sexual relationship with one partner, and proper condom use has shown to reduce the risk of adolescents developing sexually transmitted infections. On the other hand, birth control only helps to prevent pregnancy and does not offer any protection against STIs. Reirden, D. H., & Nyquist, A. (2018). Sexually transmitted infections. In W. W. Hay, M. J. Levin, R. R. Deterding, & M. J. Abzug (Eds.), Current diagnosis & treatment pediatrics (24th ed., pp. 1351-1372). McGraw Hill Education. Transdermal or vaginal estrogen avoid the risks of deep vein thrombosis and ischemic strokes. Fitzgerald, P. A. (2019). Endocrine Disorders. In M.A. Papadakis & S.J. McPhee (Eds), Current Medical Diagnosis and Treatment 2019 (58th ed., pg 1212). McGraw-Hill Education. Intermittent claudication is pain that occurs because of insufficient blood flow during times of increased demand, such as exercise. The pain is relieved with rest and is reproducible when the patient begins to walk again. Femoral pulses are usually absent or very weak and the ankle-brachial index is below 0.9. Gasper, W. J., Rapp, J. H., & Johnson, M. D. (2019). Blood Vessel & Lymphatic Disorders. In M.A. Papadakis & S.J. McPhee (Eds.), Current Medical Diagnosis & Treatment (58th ed., pp. 483-509). McGraw-Hill Education. 1 lesbians is significantly less than for heterosexual women, which creates a health disparity that needs to be corrected. While she is due for a PAP smear, the PAP with HPV co-testing is not recommended in women under the age of 30 years old. Reference: Maliver, J. O., Robertson, P. A., Ard, K. L., Mayer, K. H., & Deutsch, M. B. (2019). Lesbian, Gay Bisexual, & Transgender Health. In Papadakis, M. A., & McPhee, S. J. (Eds.) Current Diagnosis & Treatment 2019 (58th ed., pp. 1722-1742). McGraw-Hill Education.10. All of the following can help reduce the risk of adolescents developing STIs EXCEPT: A. a monogamous sexual relationship with one partner B. the use of a condom C. abstinenc e D. birth control 11. A 57-year-old female comes into the clinic with complaints of hot flashes and vaginal dryness. She is a smoker and has hypertension. When giving her hormone replacement options, the NP informs her best option is: A. Oral estrogen B. Oral estrogen with progesterone C. Estradiol injections D. Transdermal estrogen 12. When evaluating a patient with intermittent claudication, the Nurse Practitioner would expect to find all the following except: A. Diminished femoral pulses B. An ankle-brachial index of 1.0 C. Reproducible pain in the calf muscles when walking D. An ankle-brachial index of 0.8 13. A 17-year-old sexually active female presents to the clinic with complaints of vaginal pruritis, dysuria, and a thick, greenish, malodorous discharge. After further testing, it is confirmed that she is positive for Trichomoniasis. Which of the following treatment options Page 6 of 114 1 would the NP choose for this patient? A. Ceftriaxone, 250 mg IM x 1 and Azithromycin, 1 g orally as single dose Page 7 of 114 Commonly seen conditions that present with Carpal Tunnel Syndrome are pregnancy, Diabetes Mellitus, and Rheumatoid Arthritis. Luke, A., Ma, C. B., (2019) Sports Medicine & Outpatient Orthopedics. In M.A. Papadakis & S.A. McPhee (EDs)., Current Diagnosis and Treatment 2019 (5th ed., pp. 1704). McGraw Hill Education. Adolescent who suffer from ADHD, bipolar, depression, and anxiety are more associated with substance abuse. Kaul, Paritosh (2019). Adolescent Substance Abuse. In Hay, W.W., Deterding, R.R., Levin, M. J., Abzug, M.J., (Eds)., Current Diagnosis & Treatment Pediatrics (24th ed., pp. 150). McGraw Hill Education. Fluoxetine 60mg\day has been proven to break the binge\purge cycle. If Prozac is not tolerated another SSRI can be tried. Sigel, Eric (2019). Eating Disorders. In Hay, W.W., Deterding, R.R., Levin, M. J., Abzug, M.J., (Eds)., Current Diagnosis & Treatment Pediatrics (24th ed., pp. 165). McGraw Hill Education. 1 Hellmann, D. B., Imboden, J. B., (2019) Rheumatologic, Immunologic, & Allergic Disorders. In M.A. Papadakis & S.A. McPhee (EDs)., Current Diagnosis and Treatment 2019 (5th ed., pp. 844-847). McGraw Hill Education. 17. A 40-year-old woman presents to the clinic for complaints of her hands waking her up at night feeling numb with aching pain. You diagnose her with Carpal Tunnel Syndrome. All of the following are conditions do you commonly see with Carpal Tunnel Syndrome except which one? A. Pregnancy B. Diabetes Mellitus C. Rheumatoid Arthritis D. Lupus 18. Substance abuse is common in adolescent, what common comorbid condition is seen in adolescent that suffer with substance abuse? A. Bipolar disorder B. Down syndrome C. Turner’s Syndrome D. Epilepsy 19. You have an adolescent in your office who suffers with bulimia nervosa, mother states therapy did not help and wants to know if there is a medicine that can be prescribed to help the patient stop the cycle. What medication would you anticipate to start for the patient? A. (Alprazolam) Xanax B. (Amphetamine\Dextroamphetamine) Adderall C. (Fluoxetine) Prozac D. Multivitamin 20. A 32-year-old female comes into clinic with fatigue and dyspnea on exertion. She reports heavy menstrual cycles. She craves and loves to eat ice chips. What abnormal lab values would you find on this patient diagnosed with iron deficiency anemia? Page 10 of 114 1 A. Serum ferritin level 9ng/mL B. Hemoglobin 14g/dL Page 11 of 114 Ferritin value less than 12ng/mL is a high reliable indicator of reduced iron stores. A ferritin level less than 30ng/mL almost always indicates iron deficiency in anyone who is anemic. Damon, Lloyd E., Andreadis, Charalambos B (2020). Blood Disorders. In Papadakis, M. A., McPhee, S. J. and Rabow, M.W. Current Medical Diagnosis & Treatment (59th ed., pp. 515-517). McGraw Hill Education. Lachman and Anterior Drawer Tests are utilized to identify ligament laxity and are associated with a high sensitivity and specificity for ACL injury. McMurray Test is used for meniscal injury, posterior drawer test for posterior ligament injury, and the valgus stress test for collateral ligament injury to identify laxity in the MCL. Luke, A. & Ma, C. B. (2019). Sports medicine & outpatient orthopedics. In McPhee, S., Papadakis, M., & Rabow, M. (Eds.) Current Medical Diagnosis & Treatment 2019, (58th ed.). McGraw-Hill Education. (A) Unstable SCFE would present with a patient in sudden, severe pain and completely non-weightbearing in the affected leg. It is most commonly seen in obese, adolescent males. (B) Also known as Legg-Calve-Perthes Disease; most commonly presents in children 4-8 years of age. Persistent pain is the most common symptom with a limp or limited range of motion in the joint. (C) Most common in boys between ages of 12-15 due to recurrent traction on the tibial tubercle. (D) Presents with anterior knee pain, swelling, and pain. Transient Synovitis is the most common cause of limping and hip pain in children between the ages of 3-10, however it is most preceded by an upper respiratory or gastrointestinal infection. Rhodes, J., Erickson, M., Tagawa, A., & Niswander, C. (2018). Orthopedics. In Hay, W., Levin, M., Deterding, R., & Abzug, M. (Eds.) Current Medical Diagnosis and Treatment: Pediatrics 24th Ed. McGraw Hill Education 1 C. Serum ferritin level 50ng/mL D. Mean corpuscular volume (MCV): 86fL 21. A patient presents to the office with a complaint of knee pain. They report an inability to go downstairs, swelling and tenderness along the medical aspect of the knee. The nurse practitioner would perform which of the following maneuvers to assist with confirming an anterior cruciate ligament injury? A. Posterior Drawer Test B. McMurray Test C. Lachman Test D. Valgus Stress Test 22. A 6-year-old male with no recent history of illness presents to the office with a chief complaint of persistent hip pain, limping, and limited motion of the affected extremity. The provider should suspect which of the following conditions? A. Unstable Slipped Capital Femoral Epiphysis B. Avascular Necrosis of the Proximal Femur C. Osgood-Schlatter Disease D. Transient Synovitis of the Hip 23. A nurse practitioner is reviewing the completed laboratory results for a child suspicious for juvenile idiopathic arthritis (JIA). Which of the following results would support this diagnosis? Page 12 of 114 1 inquire about? A. Contaminated drinking water Page 15 of 114 Reactive arthritis most commonly occurs in young men and usually follows a gastrointestinal illness such as dysentery or sexually transmitted disease. He presents with classic symptoms: fever, ulcers, conjunctivitis, and asymmetric oligoarthritis of weight bearing joint. No gastrointestinal complaints were mentioned, patient should be screened for gonorrhea and chlamydia. Patient did not report symptoms of anxiety. Although the abdominal pain may be related to constipation, the fatigue is not relative to constipation. GERD would be described as burning pain and likely associated with meals and bedtime, fatigue would not be a symptom of GERD. The geriatric patient population will often present with somatic symptoms, such as abdominal pain, and fatigue rather than depression Essential of Diagnosis: sudden onset of neurological deficit of cerebrovascular origin. Risk factors are: HTN, DM, tobacco use, atrial fibrillation, or atherosclerosis. If stroke is suspected, a Head CT without contrast should be performed immediately, before administering aspirin or other antithrombotic agents. IV thrombolytic therapy is effective up to 4.5 hours from stroke onset. 1 B. Type of fish consumption C. Sexually transmitted diseases D. Cut or lacerations from camping trip 28. Mr. Smith is a 78-year-old male that presents to your office with complaints of intermittent, non- significant abdominal pain and fatigue. He mentions that he has just lost his wife. The FNP recognizes these complaints may indicate: A. anxiety B. constipation C. gastroesophageal reflux disease (GERD) D. depression 29. A 58-year-old male patient presents to the clinic at 1015 with complaints of right facial drooping, and right arm weakness that started around 0800. This patient has hx of obesity, hyperlipidemia, and uncontrolled diabetes. The NP is concerned about a CVA. What immediate action must be completed by the NP? A. Obtain a CBC and CMP in the clinic. B. Refer to Neurologist. C. Order a carotid US and echocardiogram. D. Send to ED immediately for STAT CT head without contrast and stroke evaluation. 30. A post-menopausal woman is in the office with complaints of low libido. She is interested in starting hormone replacement therapy (HRT) to increase her libido. Which of the following statements would be correct teaching on the risks related to HRT in post- menopausal women? A. “There are no known risk factors to HRT in post-menopausal women. Therefore, it would be an appropriate therapy for you.” B. “The risks of long-term HRT include heart attack, stroke, gallstones, breast cancer, and increased mortality. These risks are increased if you start HRT after menopause.” C. “The risks of long-term HRT include heart attack, stroke, gallstones, breast cancer, and increased mortality. These risks are reduced if HRT is started after menopause.” D. “There are risks associated with HRT, but they are minimal; therefore, the benefits outweigh the risks.” Page 16 of 114 The usefulness of a diagnostic test should not be based solely on their predictive measures (sensitivity and specificity), but also the probability that the patient has the disease. The sensitivity of a test is the ability to correctly identify those with the disease. The specificity of a test is the ability to correctly identify those without the disease. McPhee S. J., Papadakis, M. A., & Rabow, M. W. (2019). Current medical diagnosis & treatment 2019. (58th ed.). McGraw-Hill Education. In Hashimoto thyroiditis, antithyroperoxidase (TPO) antibodies will be elevated. TPO is initially checked for diagnosis and should not be routinely checked after initial diagnosis; TSH should be monitored routinely. This patient presents with hypothyroid symptoms thus we anticipate TSH to be increased and T4 to be decreased. ESR is elevated in subacute thyroiditis while antithyroid antibody titers are low. Suppurative thyroiditis both leukocyte count and ESR are usually elevated. Serum FT4 levels tend to be higher than t3 levels due to passive release of stored thyroid hormone, which is predominately T4. In autoimmune thyroiditis, the thyroid gland may be diffusely enlarged, firm, and finely nodular and they may complain of neck tightness, depression, and chronic fatigue. Fitzgerald, P. (2019). Endocrine Disorders. In M. A. Papadakis & S. J. McPhee (Eds.), Current medical diagnosis and treatment 2019 (58th ed., pp. 1132). McGraw-Hill Education. 1 Double-blind randomized control trials have shown that the risks of long-term HRT include heart attack, stroke, gallstones, breast cancer, and increased mortality rates. There is a decreased risk for these occurring if the HRT was started in menopause. It is still generally considered not safe to maintain long-term HRT therapy as the risks do not outweigh the benefits (McPhee et al., 2019). 31. A good clinician understands which of the following principles to be true about using diagnostic tests for diagnosis and management of disease? A. If a diagnostic test is positive, the patient definitely has the disease. B. If a diagnostic test is sensitive for a disease, it can correctly identify patients without the disease. C. If a diagnostic test is specific for a disease, it can correctly identify patients with the disease. D. If a diagnostic test is positive, sensitivity and specificity of the test should be considered along with the patient’s clinical presentation. 32. A 36-year-old female presents to your clinic with depression, ongoing fatigue, anterior neck tightness without pain, and weight gain. You palpate the thyroid notice goiter and a lumpy texture. There is a positive family history of thyroid problems; mother with hypothyroidism and T1DM. To work up the thyroid for this patient, what lab is specific to your potential diagnosis? A. TSH decreased B. T4 increased C. TPO elevation D. ESR decreased 33. A 9-year-old female patient presents to your clinic with her mother. She has a new onset malar rash to her face, two painless ulcers in her mouth, and is complaining of generalized joint pain. The nurse practitioner completes an assessment. Her NEXT step would be A. Order magic mouth rinse TID to help with pain when eating. B. Refer to Rheumatologist. C. Collect lab work consisting of a CBC, ESR, an ANA panel, and a UA. Page 17 of 114 1 A. Venous thromboembolic disease and stroke are increased by oral but not transdermal estrogen B. Oral estrogen increases the risk of ischemic stroke by 30% C. Long term use of oral estrogen in women over age 55 have been associated with poorer cognitive performance Page 20 of 114 Long term use of oral estrogen in women over age 65 have been associated with poorer cognitive performance. Fitzgerald, P. (2019). Endocrine Disorders. In Papadakis & McPhee, Current Medical Diagnosis & Treatment (p. 1211- 1215). McGraw- Hill Pregnancy increases the levothyroxine dosage requirement. The need for an increase in dosage has been noted as early as the first trimester (5th week). Medication increase of 20-30% is needed for healthy CNS development particularly in the second trimester. During the postpartum levothyroxine dosage typically returns to pre-pregnancy level. Decreased levothyroxine dose levels is normally required directly following delivery, bilateral oophorectomy, natural menopause, or after cessation of estrogen replacement. Fitzgerald, P. A. (2018). Endocrine Disorders. In M. A. Papadakis & S. J. McPhee (Eds.), Current medical diagnosis & treatment 2018 (57th ed., pp 1136) McGraw- Hill Education Glipizide is a Sulfonylureas with a common adverse reaction of hypoglycemia. Hypoglycemia does not occur with therapeutic doses of Metformin, Exenatide (GLP-1 receptor), and Sitagliptin (DPP-4 inhibitor). Masharani, U. (2018). Diabetes Mellitus & Hypoglycemia. In M. A. Papadakis & S. J. McPhee (Eds.), Current medical diagnosis & treatment 2018 (57th ed., pp 1236) McGraw- Hill Education 1 D. Women who receive long term estrogen therapy have increased risk for DVT 38. A 27-year-old female who is 9 weeks pregnant with a past medical history of hypothyroidism reports to the clinic in need of a Levothyroxine medication refill. You instruct the patient that A. Due to Levothyroxine’s tetroxide effects, it should be stopped and restarted after delivery of infant. B. The dose of levothyroxine should be decreased while pregnant due to slower metabolism of medication. C. She may need to increase the dose of medication after delivery of infant due to stopping medication while pregnant. D. While pregnant she will need an increased dose of Levothyroxine for adequate CNS development of fetus. 39. Mr. Frank is a 76-year-old male who presents to the clinic today with a 3-month log of FBS results. He reports many days of “low numbers” and dizziness. Which of the following medications is likely contributing to his hypoglycemia? A. Metformin B. Glipizide C. Exenatide D. Sitagliptin 40. A 26-year-old female in a strict vegan diet for several years is accompanied by her husband into your clinic with complaints of diarrhea, tingling in hands and feet, and loss of balance. The husband reports new onset of confusion. What should be at the top of the NP’s differential diagnosis based on clinical presentation? A. B12 deficiency B. Folic acid deficiency C. Iron deficiency anemia Page 21 of 114 1 D. Autoimmune hemolytic anemia Although it is rare, B12 deficiency can be seen in strict vegans. Signs and symptoms include anorexia, diarrhea, neurologic syndrome, difficulty with balance, dementia, and neuropsychiatric abnormalities. S/S of folic acid deficiency are similar, but it does not affect Page 22 of 114 The most common side effects of both typical and Atypical antipsychotics medication are anticholinergic effects such as dry mouth, delayed gastric emptying, blurred near vision, and urinary retention. Papadakis, M., McPhee, S., & Rabow, M. (2019). Psychiatric Disorders. In Current medical diagnosis & treatment (58th ed. (pp. 1063- 1118). New York, NY: McGraw Hill. It remains controversial to screen for prostate cancer using the PSA, using a CT of the lung to screen for cancer, or thyroid cancer. Lu C (2020). Benefits, risks, & costs of diagnostic tests. Papadakis M.A., & McPhee S.J., & Rabow M.W.(Eds.), Current Medical Diagnosis and Treatment 2020. McGraw-Hill. https://accessmedicine-mhmedical- A Tinel sign is tingling or shock-like pain on volar wrist percussion. The Phalen sign is pain or paresthesia in the distribution of the median nerve when the patient flexes both wrists to 90 degrees for 60 seconds. Luke, A., & Ma, C. B., (2019) Sports medicine & outpatient orthopedics. In Mcphee, S. J., & Papadakis, M. A. (2019). Current medical diagnosis & treatment (58th ed., pp 1704). Lange McGraw Hill In osteoarthritis, the joint pain is relieved by rest, unlike the pain associated with rheumatoid that is associated with morning joint stiffness. Gouty arthritis is almost always intermittent and monoarticular in the early stages but can progress into a polyarticular process in the end stage. However, gouty arthritis remains intermittent. Acute viral infections can cause polyarthritis that closely resembles rheumatoid arthritis. However, with a viral infection, fever is common, and the polyarthritis usually resolves within weeks. 1 Chesnutt, A., Chesnutt, M., Prendergast, N., & Prendergast, T. (2020). Disorders of Hemostasis, Thrombosis, and Antithrombotic Therapy. In M. A. Papadakis, S. J. McPhee, & M. W. Rabow (Authors), Current medical diagnosis & treatment 2020 (24th ed., p. 584). New York: McGraw-Hill Education. 48. All the following are side effects of using typical and atypical antipsychotics except? A. Dry mouth B. Delayed gastric emptying C. Urinary frequency D. Blurred near vision 49. Which of the following diagnostic test are appropriate preventive screening tool? A. PSA for a 60 with no urinary symptoms B. Mammogram at the age of 35 with no family history of breast cancer C. Cervical cancer screening at the age of 22 who is sexually active. D. CT of the lungs at the age of 55 with no history of smoking. 50. Your patient has a positive Tinel and Phalen sign on exam. What is your initial diagnosis? A. Olecranon Bursitis B. Carpal Tunnel Syndrome C. Dupuytren Contracture D. Medial Epicondylitis 51. A 47-year-old female patient presents to the clinic with continuous symmetric swelling of multiple joints, dry eyes and mouth, and palmar erythema. She states her joint pain and stiffness are worst in the mornings. Based on these findings, what is your diagnosis for this patient? A. Gouty arthritis B. Osteoarthritis C. Rheumatoid arthritis D. Acute viral infection P ge 25 of 114 Physical examination is normal except for “trigger points” of pain produced by palpation of areas such as the trapezius muscles, medial fat pad of the knee, and the lateral epicondyle of the elbow. Acupuncture, opioids, and corticosteroids are ineffective for pain associated with fibromyalgia and should not be used as treatment. All patients have chronic symptoms. With treatment, however, many do eventually resume increased activities. Fibromyalgia can be a complication of hypothyroidism, rheumatoid arthritis or, in men, sleep apnea. Hellmann, D. & Imboden, J. (2019). Rheumatologic, Immunologic, & Allergic Disorders. In Hay, W.W. et al. (2019). Current medical diagnosis & treatment (58th ed., pp. 888). Lange McGraw Hill. The cornerstones of PAD treatment are cardiovascular risk factor reduction and a structured exercise program. Essential elements include smoking cessation, antiplatelet therapy, lipid and blood pressure management, and weight loss. Nicotine replacement therapy, bupropion, and varenicline have established benefits in smoking cessation. Antiplatelet agents (such as aspirin, 81 mg orally daily) reduce overall cardiovascular morbidity and are recommended for all symptomatic patients. All patients with PAD should receive high-dose statin (eg, atorvastatin 80 mg daily if tolerated) to treat hypercholesterolemia and inflammation. A trial of cilostazol 100 mg orally twice a day, may improve walking distance in approximately two-thirds of patients. Supervised exercise programs for PAD provide significant improvements in pain, walking distance, and quality of life and may be more effective than an endovascular treatment alone. A minimum training goal is a walking session of 30–45 minutes at least 3 days per week for a minimum of 12 weeks. (Gasper, Rapp, & Johnson, 2019. pp.484-485). 1 Hellmann, D. & Imboden, J. (2019). Rheumatologic, Immunologic, & Allergic Disorders. In Hay, W.W. et al. (2019). Current medical diagnosis & treatment (58th ed., pp. 848-849). Lange McGraw Hill. 52. A 32-year-old female patient presents to the clinic symptoms resembling fibromyalgia. As the provider, you know all of the following to be true of this diagnosis EXCEPT: A. A normal physical exam is expected except for pain with palpation of “trigger points” B. Corticosteroids and acupuncture are first line treatments for pain associated with fibromyalgia C. Although all patients exhibit chronic symptoms, treatments can help them to increase their ability to resume daily activities D. This diagnosis can be a result of complications from hypothyroidism or Rheumatoid arthritis 53. 57 y/o male patient presents for a 3 month follow up visit, he has a history of DVT 3 months ago. He states that he is working to make lifestyle changes to prevent getting another blood clot. Which of the follow statements indicates the need for additional patient education and need for additional interventions at this visit? A. “I have really come to enjoy going to the gym. I usually go 5 days a week now.” B. “It has been really hard to quit smoking, but I only give in 1-2 times a week, which is way better than a pack a day” C. “Since we increased my blood pressure medicine last time, I have been checking it daily. Here is my log” D. “I have been taking my cholesterol medicine and blood thinner, even though I have been bruising easier I continued to take it.” 54. Your patient comes in for their annual wellness exam. Labwork reveals Hgb 10. Upon further review, your patient discloses they have an alcohol problem. You suspect their MCV will reveal which kind of anemia? Page 26 of 114 1 A. Microcytic; vitamin B deficiency B. Macrocytic; folate deficiency C. Microcytic; iron deficiency Page 27 of 114 According to Papadakis, Ch. 4., decreasing environmental hazards can help reduce risk for falls. Gait and balance training can help strengthen muscles and improve gait and balance impairment. Utilizing assistive devices correctly can help with balance, but using them wrong can increase fall risk. Canes should be used on “good” side. Avoiding inappropriate footwear, such as slippers, can help prevent accidental slips and falls. Harper, G. M., Johnston, C. B., & Landefeld, C. S. (2019). Geriatric disorders. In Papadakis, M. A., McPhee, S. J., & Rabow, M. W. (Eds), Current medical diagnosis & treatment 2019 (58th ed, pp. 55-71). McGraw-Hill. 1 Page 30 of 114 According to Papadakis, Ch. 24, migraines are usually pulsatile or throbbing on one side of the head, lasting 4-72 hours. Nausea, photophobia, and phonophobia are often accompanying symptoms. She has only tried one dose of Tylenol, so it is unlikely that this headache is related to medication overuse. Cluster headaches typically affect middle-aged men, are periorbital, and may have accompanying nasal congestion, redness of the eye, and occur daily for a certain number of hours. Tension-type headaches are described as band-like and generalized. Douglas, V. C. & Aminoff, M. J. (2019). Nervous system disorders. In Papadakis, M. A., McPhee, S. J., & Rabow, M. W. (Eds), Current medical diagnosis & treatment 2019 (58th ed, pp. 991-1062). McGraw-Hill ECT is often most effective for more severe depression and has very good results (up to 85%) in older patients with increased agitation. There are no known side effects that affect libido and serious complications occur rarely in less than 1 out of 1000 patients. Papadakis, M. A., McPhee, S. J., & Rabow, M. W. (2019). Current medical diagnosis & treatment 2019, 1059- 1116. New York: McGraw-Hill Education. According to Papadakis and McPHEE, patients on long term Warfarin should have an INR ranging between 2-3. INR less that 2 requires an increase in warfarin dosage and INR greater than 3 requires a decrease in warfarin dosage. Warfarin dose can also he held based on patients’ clinical symptoms. Leavitt, A., Minichiello, T., (2019). Disorder of Hemostasis, Thrombosis, & Antithrombotic Therapy. In M. A. Papadakis, & S.T. McPHEE. (Eds.), Current Medical Diagnosis & Treatment 2019 (58th ed., p. 583). McGraw-Hill Education. 1 61. You are seeing a 28-year-old female for a throbbing headache on the right side of her head for 2 days. She reports nausea, with her symptoms improving slightly when she stays in her dark bedroom in the quiet. She took one dose of Tylenol that didn’t really improve her symptoms much. You determine that she is most likely experiencing: A. Migraine headache B. Cluster headache C. Tension-type headache D. Medication overuse headache. 62. A 60-year-old male patient presents with history of depression complaining of suicidal ideation and increased agitation that he associates with erectile dysfunction. Patient has history of long-term use of multiple antidepressants, including his current medication being Wellbutrin and Zoloft. What would you as the provider want to include in your education about possible treatment options? A. You should stop your Zoloft today B. The Wellbutrin leads to erectile dysfunction C. We should phase you off of the Zoloft and try a tricyclic antidepressant D. We should explore options including electroconvulsive therapy 63. A patient presents to the clinic with a history of DVT. The patient states he is on Warfarin for the past 2 years. You check his INR, what range do you expect to find? A. 2.5 to 3 B. 2 to 3 C. 3 to 4 D. 2.5 to 3.5 Page 31 of 114 1 64. You have a patient who strongly believes in complementary medicine and would like more information on taking fish oil. You advise your patient that fish oil has a positive effect on all of the following except which? Page 32 of 114 Michael’s laboratory results meet the diagnostic criteria for prediabetes. Starting Metformin once reaching prediabetic threshold will prevent/delay progression to T2DM. Michael is also obese and should also be educated to start a Mediterranean diet, and to increase his exercise to 150min/week. Obesity is the common cause for developing insulin resistance. Michael should continue to be screened for the development of T2DM annually. Masharani, U. (2019). Chapter 27: Diabetes Mellitus. In Papadakis, M. A., McPhee, S. J., & Rabow, M. W (Eds.), Current Medical Diagnosis & Treatment, 2019 (pp 1220-1266) New York, NY: McGraw-Hill Education. Diabetic neuropathy is the most common complication of T2DM. Treatment should start with TCAs such as Nortriptyline; however, TCAs are on the beers criteria and not appropriate for a 78-year-old patient. Pregabalin is not used as first line treatment due to its abuse potential. Pregabalin is a Schedule V drug. Fluoxetine is not a treatment of diabetic neuropathy. Masharani, U. (2019). Chapter 27: Diabetes Mellitus. In Papadakis, M. A., McPhee, S. J., & Rabow, M. W (Eds.), Current Medical Diagnosis & Treatment, 2019 (pp 1220-1266) New York, NY: McGraw-Hill Education. Prevention is the most important educational focus when it comes to avoiding detrimental neurological events. Blood pressure control is a modifiable risk factor and therefore takes priority when trying to educate patients about TIAs and strokes. Though being elderly, female, African American, having a personal history of TIA, and having a family history of stroke are risk factors for the patient, they are non-modifiable and out of the patient’s control. 1 Papadakis, M. A., McPhee, S. J., & RaBow, M. W. (2019). Endocrine Disorders. In P.A. Fitzgerald (Ed.), Current: Medical diagnosis & treatment (58th ed. (pp. 517-518). New York, NY: McGraw-Hill Education. 68. Michael, a 45-year-old male, with a BMI of 30 presents to your clinic for a routine wellness exam. He completed his lab work prior to his appointment with the following results: Hgb A1C 6.1%, eGFR 55 mL/min, and a fasting glucose of 123 mg/dl. Based on these results what is the most appropriate action by the provider? A. No intervention necessary, these results are considered normal follow up in 1 year. B. Refer to Endocrinologist. C. Start Metformin and encourage dietary and lifestyle changes. D. No intervention necessary, recheck A1C in 4 months 69. You see Henry, a 78-year-old male with T2DM with complaints of numbness and tingling in his feet. You diagnose Henry with diabetic neuropathy. Which of the following medications is considered first line treatment for diabetic neuropathy in this patient population? A. Nortriptyline B. Gabapentin C. Pregabalin D. Fluoxetine 70. An elderly, African American, female patient who has recently experienced a transient ischemic attack is trying to inquire more information about strokes. As the health care provider, which stroke education statement would be the most important information to convey to the patient? A. “Blood pressure control is an important factor when trying to prevent future strokes.” B. “Being an elderly, African-American, female puts you at an increased risk for a stroke.” C. “Having a history of a TIA puts you at an increased risk for stroke.” D. “Having a family history of stroke or TIA can put you at an increased risk for both.” Page 35 of 114 Pain is usually worse with back extension and relieved by sitting in spinal stenosis. Lumbar disk herniation is usually due to bending or heavy lifting with the back in flexion and pain is caused by back flexion or prolonged sitting. Nerve root impingement can be caused by injury or compression or neural structures and causes “claudication” symptoms and radicular pain with ambulation. OA is a degenerative change in joint cartilage and causes pain with loading of the joint or at the extremes of motion. Subclinical hypothyroidism is defined as having a normal T4 and an elevated TSH above reference range. The TSH will often normalize over time and does not require treatment if the patient is asymptomatic. Fitzgerald, P. A. (2018). Endocrine Disorders. In W. W. Hay, R. R. Deterding, M. J. Levin, & M. J. Abzug (Eds.), Current diagnosis & treatment, Pediatrics (24th ed., pp. 1136). McGraw-Hill Education. A, B, and D are all correct. The vegan or vegetarian diet precludes consumption of heme (animal source) iron. Nonheme iron is 1- 5% absorbed vs. 10-20% absorption of heme iron. Microcytic RBCs will be observed rather than macrocytic. Macrocytic cells are seen in vitamin B12 deficiency which occurs more often with alcoholism (this patient does not drink), folate deficiency, and DNA synthesis inhibitors. Reduced iron absorption and sources due to a vegan diet, and heavy menstrual bleeding indicate iron deficiency anemia. 1 Douglas V.C., & Aminoff M.J. (2020). Part 24-08: Stoke. In M. Papadakis, & S. McPhee, & M. Rabow (Eds.), Current medical diagnosis & treatment 2020. McGraw-Hill. Retrieved from https://accessmedicine-mhmedical- com.libproxy.usouthal.edu/content.aspx?bookid=2683&sectionid=225131316 71. Ms. Jane is a 50-year-old female who presents to clinic with complaints of mid lower back pain that started a few days ago after working in her yard. She states the pain is worsened by bending forward and sitting down for long periods. You would expect to find what on her imaging study? A. Spinal stenosis B. Osteoarthritis C. Lumbar disk herniation D. Nerve root impingement 72. Which of the following lab values would a provider expect to find in subclinical hypothyroidism? A. TSH elevated, T4 normal B. TSH decreased, T4 normal C. TSH elevated, T4 decreased D. TSH decreased, T4 elevated 73.The APRN sees a 36-year-old female patient complaining of fatigue, shortness of breath and shortness of breath with exertion. She has no medical history, denies any medications or alcohol abuse. She reports an extremely healthy vegan lifestyle that incorporates regular cardiovascular exercise and a balanced diet. She reports heavy periods, with significant flow. The APRN knows all the following to be true about anemia EXCEPT: A. The most common cause of anemia is iron deficiency B. Bleeding is the most common cause of iron deficiency in adults C. Macrocytic RBC will likely be observed on the CBC. D. A vegan diet supplies only nonheme iron that is poorly absorbed, and not likely to meet nutritional needs. Page 36 of 114 1 74. What electrolyte disturbance would you most commonly find with bulimia nervosa? A. Metabolic alkalosis, hypokalemia, hypochloremia, hyponatremia Page 37 of 114 Side effects of statins include fatigue, elevated liver enzymes and muscle pain. When patients present with these symptoms, the drug should be discontinued, and liver enzymes should be checked to assess the effects on the liver. Simvastatin is associated with the highest risk of muscle injury and pain. Baron, R.B. (2019). Lipid Disorders. In M.A. Papadakis & S.J. McPhee (Eds.), Current Medical Diagnosis & Treatment (58th ed., pp. 1267-1278). McGraw-Hill Education. Because the chest pain was brought on by physical exertion and immediately relieved with rest, the pain is consistent with stable angina pectoris. The patient should see a cardiologist to rule out any potential cardiac problems and to ensure that physical exercise is safe. Bashore, T. M., Jackson, K. P., & Patel, M. R. (2019). Heart Disease. In M.A. Papadakis & S.J. McPhee (Eds.), Current Medical Diagnosis & Treatment (58th ed., pp. 2019-). McGraw-Hill Education. Acute Bacterial Prostatitis presents with fever, irritative voiding symptoms, perineal and/or suprapelvic pain, and a positive UA. Chronic bacterial prostatitis does not present with fever or a positive UA. Acute epididymitis will present with painful scrotal swelling due to enlargement of the epididymitis. Prostatodynia is a noninflammatory disorder presenting with a normal UA and no fever (Papadakis & McPhee, 2019, p. 970-973). Papadakis, M.A., McPhee, S.J. (2019). Urologic Disorders. In M.V., Wang, T.J., Walsh, & T.D., Chi (Eds.), Current medical diagnosis & treatment (58th ed., pp. 970 - 973). United States of America: McGraw-Hill Education. 1 4. A 52-year-old female who has been taking pravastatin 20 mg nightly for the past 6 months, comes into the clinic complaining of extreme fatigue, dark urine, and muscle pain for the past 3 weeks. Which of the following is the best treatment plan for the NP to follow? A. Continue the pravastatin but at half the dose B. Recommend increasing fluid intake and rest. C. Discontinue the pravastatin and order a liver function panel D. Switch to Simvastatin 40 mg nightly 5. A 45-year-old male patient comes to clinic after an episode of chest pain in his substernal area that occurred while he was running. He says that the pain was relieved immediately when he stopped running. Which of the following should the NP recommend to this patient? A. See a cardiologist for further evaluation. B. Take 600 mg ibuprofen every 6 hours as needed for pain. C. Begin an exercise program that includes walking instead of running. D. Consult a gastroenterologist to rule out gastroesophageal reflux disease. 6. A 23-year-old male patient presents to the clinic with complaints of fever, irritative voiding symptoms, and perineal/suprapubic pain for 2 days. On exam, the CBC shows leukocytosis and a left shift. UA is positive for pyuria and bacteriuria. There is no CVA tenderness or painful scrotal enlargement. The NP suspects the patient has which of the following diagnosis? A. Acute Bacterial Prostatitis B. Chronic Bacterial Prostatitis C. Acute Epididymitis D. Prostatodynia Page 40 of 114 Oral androgen therapy with methyltestosterone is not recommended because it can produce acute hepatitis and chronic high-dose use can cause peliosis hepatis, cholestatic hepatitis, and hepatocellular carcinoma. All the other choices are appropriate treatment options (Papadakis & McPhee, 2019, p. 1199-1203.) Papadakis, M.A., McPhee, S.J. (2019). Endocrine Disorders. In P.A. Fitzgerald (Eds.), Current medical diagnosis & treatment (58th ed., pp. 1199-1203). United States of America: McGraw-Hill Education. For patients with CAP and comorbid medical conditions who recently had antibiotic use in the last 3 months require a respiratory Fluoroquinolone or a Macrolide plus a Beta Lactam antibiotic for coverage. Chesnutt, A.C., Chesnutt, M.C., Prendergast, N.T., Prendergast, T.J., (2019) Pulmonary Disorders. In M.A. Papadakis & S.A. McPHee (Eds)., Current Diagnosis and Treatment 2019 (5th ed., pp. 279). McGraw Hill Education. Foreign body aspiration should always be suspected when you have a child who presents with a chronic cough, persistent wheezing, recurrent pneumonia and auscultating asymmetrical breath sounds. Federico, M.J., Baker, C.D., Deboer, E.M., Halbower, A.C., Martiniano, S.L., Sagel, S.D., Stillwell, P., Zemanick, E.T., Caraballo, M., Hawkins, S., (2019). Respiratory Tract & Mediastinum. In Hay, W.W., Deterding, R.R., Levin, M. J., Abzug, M. J., (Eds)., Current Diagnosis & Treatment Pediatrics (24th ed., pp. 525). McGraw Hill Education. 1 7. A 45-year-old healthy male presents to the clinic with complaints of fatigue, decreased libido, and reduced exercise endurance. The NP diagnoses the patient with Hypogonadotropic Hypogonadism after obtaining lab results showing a low testosterone level, normal FSH, LH, and prolactin levels. All the following are appropriate treatment options for this patient EXCEPT: A. Testosterone topical gels B. Transdermal testosterone patches C. Oral methyltestosterone supplementation D. Buccal testosterone tablets 8. A 68-year-old white female presents to a clinic with a 2-week history of fever up to 101, productive cough, and congestion. She has a history of asthma and type 2 diabetes. She explains she was hospitalized one month ago for pneumonia. Chest x-ray today revealed pulmonary opacity in the RUL. Which treatment option would be best for this patient? A. Amoxicillin 500mg po BID x 10 days and return for follow up in one month for repeat chest x-ray. B. Azithromycin 500mg orally as a first dose then 250mg orally daily for 4 days. C. Azithromycin 500mg orally as a first dose then 250mg orally daily for 4 days plus Amoxicillin 1G orally TID x 10 10 days. D. Cephalexin 500mg po BID x 10 days and follow up in 2 weeks. 9. A 3-year-old presents to your clinic with a sudden onset of coughing, wheezing, and respiratory distress. All of the following would make you suspicious of a foreign body aspiration EXCEPT which one? A. Hearing symmetric breath sounds B. Hearing asymmetric breath sounds C. Persistent wheezing D. Recurrent pneumonia Pag 41 of 114 According to the USPSTF guidelines you do not start cytology screening until a female is 21 years old and then every 3 years if cytology is normal. Reference: Sass, A., & Richards, M. (2018). Adolescence. In W.W. Hay, M.J. Levin, R.R. Deterding, & M.J. Abzug (Eds.), Current diagnosis & treatment pediatrics (24th ed., pp. 261). McGraw-Hill Education A lipid profile and hematocrit should be closely monitored because testosterone replacement can cause hyperlipidemia and erythrocytosis. Long-standing or severe hypogonadism can cause osteoporosis and the patient has had a recent fracture so a bone densitometry scan should be ordered. McPhee S. J., Papadakis, M. A., & Rabow, M. W. (2019). Current medical diagnosis & treatment 2019. (58th ed.). McGraw- Hill Education. Oral contraceptive and estrogen replacement can commonly cause a clear, milky, or serous nipple discharge from single and/or multiple ducts usually according prior to menses. A side effect of anti-psychotics is milky nipple discharge. This can also be a symptom of undiagnosed hypothyroidism and warrants a TSH. The exam was negative for signs of breast abscess. 1 10. A 15-year-old sexually active patient comes into your clinic for a wellness exam. What screenings are NOT appropriate to perform on this patient? A. Cervical cancer B. Hypertension C. Depression/anxiety D. Obesity 11. A 66-year-old male patient with a history of hypogonadism presents to the clinic for a follow up visit and medication refill of testosterone cypionate 200 mg intramuscularly every two weeks. The patient admits to falling two months ago and sustained a fracture to his left distal fibula. He reports increased energy level, less depressed mood, and improvement in libido. His most recent testosterone level was 199 and PSA 1.1 six months ago. In conclusion of today’s visit, the nurse practitioner will order all of these EXCEPT? A. Testicular ultrasound B. CBC C. Bone densitometry scan D. Lipid Panel 12. A healthy appearing 30-year-old female presents to the clinic for her annual wellness exam with Pap today. The patient mentions having intermittent episodes of bilateral milky appearing nipple discharge for two months. Her family history is negative for breast and ovarian cancers. Clinical breast exam reveals a scant amount of milky discharge, skin color and texture are normal, non-tender, without dimpling, retraction, or mass. The nurse practitioner considers all EXCEPT one as a possible cause for nipple discharge. A. Oral contraceptives B. Breast abscess C. Anti-psychotic medications D. Hypothyroidism Page 42 of 114 Alkaline Phosphatase is an enzyme that can be found in your liver, digestive system, kidneys and bones. If your liver is not functioning properly, ALP will be elevated. Vimalraj, S. (2020). Alkaline phosphatase: Structure, expression and its function in bone mineralization. Gene, 754, 144855. https://doi-org.libproxy.usouthal.edu/10.1016/j.gene.2020.144855 NAAT test is the most sensitive test in detecting specific STD’s (Gonorrhea, Chlamydia & Herpes Simplex), however it is not available in every lab. Therefore, there are other tests that may be less sensitive, but still able to detect these common STD’s. Brook, G. (2015). The performance of non-NAAT point-of-care (POC) tests and rapid NAAT tests for chlamydia and gonorrhea infections. An assessment of currently available assays. Sexually Transmitted Infections, 91(8), 539–544. Most common clinical findings of croup are URI symptoms followed by barking cough and stridor. An AP and lateral neck x-ray shows subglottic narrowing or the steeple sign without irregularities seen in the tracheitis and normal epiglottitis. The presence of cough and the absence of drooling favor the diagnosis of viral croup over epiglottitis. Federico, M. J., Baker, C. D., Deboer, E. M., Halbower, A. A., Kupfer, O., Martiniano, A. L., Sagel, A. D., Stillwell, P., Zemanick, E. T., Caraballo, M., & Hawkins, S. (2018). Respiratory tract & mediastinum. In W. W. Hays, M. J. Levin, R. R. Deterding, & M. J. Abzug (Eds.), Current diagnosis & treatment: Pediatrics. McGraw- Hill. 1 D. Transient Synovitis is the most common cause of limping and hip pain in children between the ages of 3-10, however it is most commonly preceded by an upper respiratory or gastrointestinal infection. Rhodes, J., Erickson, M., Tagawa, A., & Niswander, C. (2018). Orthopedics. In Hay, W., Levin, M., Deterding, R., & Abzug, M. (Eds.) Current Medical Diagnosis and Treatment: Pediatrics 24th Ed. McGraw Hill Education 17. Karen, a 52-year-old female, presented to the clinic for a suspicious lump in her right breast. After mammography, imaging and a biopsy, Karen is diagnosed with breast cancer. Which lab finding could be indicative of metastases to the liver or bone? A. Hypercalcemia B. Serum Alkaline Phosphatase C. CA 15-3 marker D. CA 27-29 marker 18. Robin comes into the clinic with concerns of an STD. She is having vaginal discharge and dysuria. As a Nurse Practitioner, you know that the NAAT test can be performed by a cervical/vaginal swab or first- void urine. Which STD is NOT sensitive to the NAAT test? A. Chlamydia B. Gonorrhea C. Herpes Simplex D. Syphilis 19. The following clinical findings are consistent with the diagnosis of Croup except: A. Drooling B. Stridor C. Barking cough D. Subglottic narrowing (steeple sign) on neck x-ray Page 45 of 114 With COPD, pulmonary function testing shows airflow obstruction that is not fully reversible and is most often progressive. Asthma is correlated with airflow obstruction that is reversible. Chesnutt, A. N., Chesnutt, M. S., Prendergast, N. T., Prendergast, T. J., (2019). Pulmonary disorders. In M. A. Papadakis & S. J. McPhee (Eds.), Current Medical Diagnosis & Treatment 2019 (5th ed., pp. 253-268). McGraw-Hill Education. Acute Inflammatory Pericarditis is usually seen with viral infections like hepatitis, varicella, mumps, HIV, etc. It’s characterized by pericardial friction rub, fever, elevated ESR and CRP, ECG with ST segment elevation with PR depression, and anterior pleuritic chest pain that is worsening when lying flat. Bashore, T. M., Granger, C. B., Jackson, K. P., Patel, M. R., (2019). Heart disease. In M. A. Papadakis & S. J. McPhee (Eds.), Current Medical Diagnosis & Treatment 2019 (5th ed., pp. 437). McGraw-Hill Education. Secondary prevention requires identifying and treating sexually transmitted infections before they occur. P. 1352. The first pap should be performed at age 21 and then every 3 years. HPV typing is not recommended before age 30. P. 1353 Hay, W. W. et al. (2018). Chapter 44: Sexually Transmitted Infections. In Reirden, D.H., MD & Nyquist, A. C., MD, MSPH (Eds.), Current diagnosis & treatment pediatrics (24th ed., pp.1352-53). Lange McGraw Hill. 1 20. A 53-year-old male presents to the clinic for evaluation of cough and shortness of breath over the past three months. He undergoes pulmonary function testing that reveals airflow limitation that is not fully reversible. As the provider, you suspect? A. Asthma B. Pneumonia C. TB D. COPD 21. 34-year-old AAM with history of HIV presents with fever, pleuritic chest pain that is worse when supine, elevated sedimentation rate and CRP, and pericardial friction rub. What is the most likely diagnosis? A. Endocarditis B. Pneumonia C. Acute inflammatory Pericarditis D. Tuberculosis 22. Identify the example of secondary prevention. A. Performing a pap smear on a 23-year-old B. Immunizing a 15-year-old with Gardasil C. Treating Pelvic inflammatory disease D. Discussing the use of condoms with an adolescent patient 23. Which of the following is a secondary condition known to affect lipid metabolism? A. Alcohol use B. Coronary heart disease C. Hypertension D. Congestive heart failure Page 46 of 114 The use of Niacin can cause “hot flashes” or pruritus. Flushing can be decreased with the use of aspirin or other NSAIDs. Flushing can also be decreased by starting Niacin in a low dose taken with the evening meal. Papadakis, M., McPhee, S., & Rabow, M. (2019). Lipid disorders. In Current medical diagnosis & treatment (58th ed. (pp. 1267-1275). New York, NY: McGraw Hill. Signs and symptoms of early breast cancer is a single, non-tender firm to hard mass (749). Late findings are nipple or skin retraction, and breast enlargement (749). A sign of advance breast cancer is edema in the ipsilateral arm due to metastic infiltration of regional lymphatics (754). Advance diagnosis is expressed by axillary nodes that are matted or fixed to skin or deep structures (753). Giuliano, A. E., & Hurvitz, S. A. (2018) Breast Disorders. In W.W. Hay, M. J. Levin, R.R. Deterding, & M. J. Abzug Trichomonas is treated with Metronidazole or Tinidazole 2G orally as single dose or 500mg orally bid for 7days. (776). Fluconazole is to treat vulvovaginal candidiasis (776). Bacterial Vaginitis is treated with Metronidazole 500mg orally bid for 7days or Clindamycin 300mg orally bid for 7days (777) Woo, J. (2018) Gynecologic Disorders. In W.W. Hay, M. J. Levin, R.R. Deterding, & M. J. Abzug (Eds.), Current diagnosis & treatment pediatrics (24th ed., pp.770-804). McGraw Hill Education. 1 Excessive alcohol use can increase triglyceride levels. A reduction in alcohol use has been lipid-lowering therapy. Papadakis, M., McPhee, S., & Rabow, M. (2019). Lipid disorders. In Current medical diagnosis & treatment (58th ed. (pp. 1267-1275). New York, NY: McGraw Hill. 24. Which of the following medication can cause pruritis? A. Atorvastatin (Lipitor) B. Omega-3-acid ethyl esters (Lovaza) C. Niacin (Niaspan) D. Colesevelam (Welchol) 25. When performing a breast exam on a 63-year-old women, you find a new mass. In diagnosing breast cancer, you know that the following is associated with early breast cancer. a) erythema and edema of the ipsilateral arm b) single painless firm lump c)skin or nipple retraction d)fixed or matted axillary nodes 26. You are seeing a 27 year old who is sexually active with complaints of a discharge for several weeks. Further investigation, you learn she is experiencing itching, frothy- green discharge, and via speculum exam red macular lesions on cervix. What is the medication recommended upon your diagnosis? A. Metronidazole 2G orally one time dose B. Fluconazole 150mg orally one time dose C. Clindamycin 300mg orally bid for 7 days D. Ampicillin 500mg orally bid for 5 days Page 47 of 114 Women should gain weight during their pregnancy to ensure normal fetal growth and development. This woman should be encouraged to gain less weight than a woman with a healthy BMI. A woman with a healthy BMI should gain 25-35lbs during pregnancy, an overweight woman should gain 15-25lbs, and the obese woman should limit weight gain to 11-20lbs. Option A is the only correct option. Bedtime administration of levothyroxine results in high Free T4 and lower TSH levels than morning administration. Absorption of levothyroxine can be affected by coadministration of other drugs. Labs will be rechecked in 4-6 weeks to determine the need for dose titration. Levothyroxine dosage will often need to be increased during pregnancy. Patients need to consistently take the same manufacture’s brand of levothyroxine due to the variation of bioavailabity of levothyroxine preparations. Fitzgerald, P. A. (2019). Endocrine disorders. In. M. A. Papadakis, S. J. McPhee, & M. W. Rabow (Eds.), Current medical diagnosis & treatment 2019(58th ed., pp. 1119- 1219). Options A, B, and C are all correct. Discontinuing or reducing the use of corticosteroids should be done if possible. Smoking cessation and reduction in alcohol intake is encouraged. Walking and resistance exercise can increase bone density and strength thus reducing risks of fractures due to falls. Diets should be high in protein, calories, calcium, and Vitamin D. Fitzgerald, P. A. (2019). Endocrine disorders. In. M. A. Papadakis, S. J. McPhee, & M. W. Rabow (Eds.), Current medical diagnosis & treatment 2019(58th ed., pp. 1119- 1 31. An obese (BMI >30) pregnant woman presents to your office. She is 22 weeks pregnant and has gained 10lbs. As a provider, it is important that you educate your obese patient that she should gain no more than pounds during the course of her pregnancy to help ensure a normal birth weight infant and healthy infant. A. 25-35lbs B. 15-25lbs C. 11-20lbs D. Lose 15lbs 32. A 25-year-old female patient presents to the clinic with symptoms of weight gain, constipation, and fatigue. Labs are unremarkable except for TSH of 10.12. What education about the treatment plan is important for the patient to know? A. Take your medication first thing in the morning on an empty stomach. B. We will recheck your labs in two weeks. C. There is no need to notify your medical provider if you become pregnant. D. All levothyroxine preparations have the same bioavailablity so is it ok to switch between brand manufactures. 33. A female patient’s bone density scan results in a T-score of -1.5. All of the following are interventions to help prevent osteoporosis EXCEPT: A. Reduce the use of corticosteroids. B. Encourage smoking cessation. C. Increase exercise regimen. D. Eat a diet low in Vitamin D. Page 50 of 114 Peripheral pulmonary artery stenosis is often heard in newborns to the upper left sternal border. A still murmur, though the most common is heard loudest between the apex and left lower sternal border. Pulmonary ejection murmur is also heard on the left side of the heart and is heard the loudest when the patient is supine. A venus hum is heard after the age of 3 and heard loudest while the child is sitting and goes away when the child lies supine. Jone, P., VonAlvensleben, J., Bukett, D., Darst, J. R., et. al. (2018). Cardiovascular diseases. Hay, W. W., Levin, M. J., Respiratory distress is one of the most common complexes of symptoms in a newborn. This can, but not always, be related to cardiopulmonary causes. The most common clinical features present for diagnosis include Tachypnea with a respiratory rate of greater than 60 breaths/minute, expiratory grunting, cyanotic on room air, and sternal/intercostal retractions. Circumcision is elective and can be performed in healthy infants who are stable. Medical benefits include prevention of UTI’s, decreased risk of penile cancer, decreased incidence of contracting STI’s and HIV. Smith D, & Grover T (2018). The newborn infant. Hay, Jr. W.W., & Levin M.J., & Deterding R.R., & Abzug M.J.(Eds.), Current Diagnosis & Treatment: Pediatrics, 24e. McGraw-Hill. https://accesspediatrics-mhmedical- com.libproxy.usouthal.edu/content.aspx?bookid=2390&sectionid=189072322 1 34. A 4-year-old is in the clinic today for a well-child check. On auscultation of the infraclavicular area on the right, you hear a constant musical hum while the child is sitting. You ask the child to lie down and auscultate the same location and you no longer hear a hum. Which innocent murmur is this? A. Still murmur B. Pulmonary ejection murmur C. Venus hum D. Peripheral pulmonary artery stenosis 35. When performing an assessment for an infant with suspected respiratory distress, an NP would expect to see all of the following clinical features EXCEPT: A. Intercostal & sternal retractions B. Expiratory grunting C. Respiratory rate of 26 breaths per minute D. Cyanosis in room air 36. A pregnant mother in your clinic inquires about circumcision post-birth. In providing risk and benefits surrounding this procedure, you tell her that: A. There are no known benefits, it is preference only. B. This procedure can prevent urinary tract infections C. This can procedure has a high incidence of complications D. This procedure is associated with increased risk of contracting STI’s later in life 37. Which of the following is not a reason for high-intensity statin therapy? A. LDL greater than 190 B. Age 65 with presence of atherosclerotic cardiovascular disease Page 51 of 114 1 C. 40-year-old diabetic with LDL of 140 with CVD risk of 10% D. 45-year-old diabetic with LDL 75 with CVD risk less than 7.5% Page 52 of 114 A patient at risk for drug resistance (antibiotic tx within 90 days, greater than 65 y.o., exposure to child in daycare, immunosuppression, comorbid illness) should receive a respiratory fluoroquinolone as recommended treatment. Chestnut, A. N., Chesnutt, M. S., Prendergast, N. T., & Prendergast, T. J. (2018). Pulmonary Diorders. In W. W. Hay & ACE inhibitors, NSAIDS, Sulfonamides, and Angiotensin-11 receptor blockers are a few of many drugs contraindicated in pregnancy due to their known fetotoxic/teratogenic effects. Methyldopa, along with medications such as nifedipine and labetalol, have been proven safe during pregnancy. Acetaminophen products are advised for pain during pregnancy. Rogers, V.L., & Roberts, S.W. (2019). Obstetrics and obstetrics disorders. In M.A. Papadakis & S.J. McPhee (Eds.), Ectopic pregnancy usually occurs in the first trimester of pregnancy. Clinical signs and symptoms include severe, intermittent, sudden abdominal pain that does not radiate. Cholelithiasis, urinary calculi, and constipation are all possible but are not emergent. Rogers, V.L., & Roberts, S.W. (2019). Obstetrics and obstetrics disorders. In M.A. Papadakis & S.J. McPhee (Eds.), Current Medical Diagnosis & Treatment 2019 (5th ed., pp. 811-839). McGraw-Hill Education. 1 A. Azithromyc in B. Levofloxacin C. Amoxicillin D. Cefuroxime 42. All of the following medications are known to have fetotoxic or teratogenic effects and are contraindicated in pregnancy EXCEPT: A. Losartan B. Sulfamethoxazole/trimethoprim C. Ibuprofen D. Methyldopa 43. A 24-year-old healthy female arrives at the outpatient OBGYN clinic for her 8-week routine pregnancy visit. She is complaining of sudden, severe, intermittent lower left quadrant pain that began two days ago. The pain does not radiate. The NP should consider which of the following as a potential cause that requires immediate evaluation and intervention? A. cholelithiasis B. ectopic pregnancy C. urinary calculi D. pregnancy-induced constipation 44. When providing care for all MSM, the Nurse Practitioner will order all of the following routine screening tests at least annually ( or more often if indicated) EXCEPT: A. Syphilis serology Page 55 of 114 1 B. HIV serology C. Anal cytology D. NAAT for gonorrhea & chlamydia Page 56 of 114 Asking these important questions about sexuality and how your patients identify are crucial in the patient/NP relationship. This opens dialog for patients and helps them feel heard and valued. It also encourages ongoing care as they feel accepted in your practice. Knowing why they have not been to a healthcare facility in several years may have importance, but knowing their sexuality is the most important. Obedin-Maliver, J., Robertson, P.A., Ard, K.L., Mayer, K.H. & Deutsch, M.B. (2018). Lesbian, gay, bisexual, & transgender health. In W.W. Hay, M.J. Levin, R.R. Deterding, & M.J. Abzug (eds). Current diagnosis & treatment pediatrics (24th ed., pp 1731-1749). McGraw Hill Education. Bisexual men feel they are outcast, not only with heterosexual men, but their gay counterparts as well, as they don’t “fit in” with either group making support difficult. They also often hold a stigma to themselves, predisposing them to mental illness. Bisexual men are not confused about who they are attracted to. While they are a minority, this does not place them at a higher risk for mental illness, neither does being male. 1 Current clinical practice guidelines recommend routine annual screening for syphilis, HIV, and gonorrhea/chlamydia in all MSM. Anal cytology screening is recommended for MSM who are HIV positive. Obedin-Maliver, J., Robertson, P.A., Ard, K.L., Mayer, K.H., & Deutsch, M. B. (2019). Lesbian, gay, bisexual, & transgender health. In M.A. Papadakis & S.J. McPhee (Eds.), Current Medical Diagnosis & Treatment 2019 (5th ed., pp. 1722-1742). McGraw-Hill 45. The Nurse Practitioner knows all of the following are health disparities affecting bisexual and lesbian women EXCEPT: A. Asthma B. Alcohol and tobacco abuse C. Cardiovascular death D. Anorexia Asthma, substance abuse, obesity (not anorexia), and cardiovascular death are all health disparities found in the bisexual & lesbian population. Obedin-Maliver, J., Robertson, P.A., Ard, K.L., Mayer, K.H., & Deutsch, M. B. (2019). Lesbian, gay, bisexual, & transgender health. In M.A. Papadakis & S.J. McPhee (Eds.), Current Medical Diagnosis & Treatment 2019 (5th ed., pp. 1722-1742). McGraw-Hill Education. 46. You are seeing a 28-year-old female patient for the first time. She has no previous visits or medical history found in your EMR. One of the most important questions to ask during your initial history interview is: A. “How do you identify sexually? Do you consider yourself lesbian, bisexual, heterosexual or something else?” B. “Why haven’t you been to a doctor in the past several years?” C. “Have you been seen in this office before?” D. “What’s your favorite recreational activity?” 47. Bisexual men are at a higher risk for mental illnesses, such as depression and/or anxiety. The nurse practitioner knows this is due to: A. Their confusion as to what gender they are attracted to. B. Them being a minority group in the United States. C. Stigmatization from heterosexual and gay men, along with their own stigmatization of their selves. D. Them being the male gender. Page 57 of 114 When diagnosing a patient with hypertension there are many things a provider must ask about in order to figure out the cause. Some exacerbating factors include: weight (BMI>30), high alcohol consumption, cigarette smoking, low potassium intake, NSAID therapy, polycythemia, high salt intake, and sleep apnea. Sutters, M. (2019). Systemic hypertension. In M. A. Papadakis & S. J. McPhee (Eds.), Current medical diagnosis & treatment (pp. 451- 482). McGraw Hill Education. In tetralogy of Fallot an infant will present with hypoxemic spells or “Tet spells” that present around 4-6 months and normally seen while the infant is eating or crying. Lab findings reveal elevated hemoglobin, hematocrit, and RBC due to chronic desaturation. In a chest radiograph the infant will show a normal size heart but will show some right ventricular hypertrophy and an upturning of the apex. Jone, P., VonAlvensleben, J., Burkett, D., Darst, J. R., Collins, K. K., & Miyamoto, S. D. (2018). Cardiovascular diseases. In W. W. Hay, Appendicitis is consistent with the patient’s right lower quadrant tenderness but not with high levels of β-human chorionic gonadotropin (β-hCG). The cholecystitis pain is in a different location; it is usually in the middle or right side of your upper abdomen. It may also spread to your right shoulder or back. The pain in pancreatitis is in a different location, and it is usually located on the upper abdominal area just under the ribs; it can radiate to the back, flank, or chest. 1 51. You have just diagnosed a 40-year-old female with hypertension and are trying to figure out any exacerbating factors that might be contributing to this new diagnosis. What information below would not be pertinent to make your decision? A. BMI and diet habits B. Alcohol and tobacco use C. Water consumption D. Diagnosis of sleep apnea 52. Your next patient is a 6-month-old boy who has recently adopted from Haiti by his family. You suspect he has tetralogy of Fallot based on which of the following clinical findings except: A. An enlarged heart on radiograph B. Thickening of the right ventricular wall C. Elevated hemoglobin, hematocrit, and RBC D. Cyanotic spells while eating or crying 53. An 18-year-old female G0P0 complains of nausea, vomiting, and abdominal pain for three days. Her last menstrual period was 6 weeks ago. Her menarche was at 12-years-old and is sexually active. She is not taking oral contraceptives and uses condoms infrequently. She is afebrile, with a blood pressure of 113/76 and a heart rate of 95 BPM. Physical examination is significant for tenderness of the right lower quadrant during palpation. Laboratory tests show that the β-human chorionic gonadotropin (β- hCG) level is 2500 mIU/L. Transvaginal ultrasonography reveals no intrauterine pregnancy. What is the most likely diagnosis? A. Appendicitis B. Cholecystitis C. Pancreatitis D. Ectopic pregnancy Page 60 of 114 The most significant evidence of placental abruption is the presence of painful vaginal bleeding, which is absent in this patient. Myasthenia gravis is a long-term neuromuscular disease that leads to varying degrees of skeletal muscle weakness. Miscarriage is when an embryo or fetus dies before the 20th week of pregnancy, and fluid, blood, or tissue passing from the vagina is usually present. Preeclampsia is characterized by new onset of hypertension and proteinuria during pregnancy. Papadakis, M. A., McPhee, S. J., Rabow, M. W., Rogers, V. L., & Roberts, S. W. (2020). Obstetrics & Obstetric According to developmental milestones, at the age of 30 months children begin to learn how to hop on 1 foot. Temper tantrums occur between the ages 1-4 years old and commonly occur in this age group to cope with their feelings and achieve autonomy. Between ages 2-3 children engage in parallel play. Its not until the age of 3-4 that they engage in interactive play. Early characteristics of Autism Spectrum Disorder are the inability to use gestures and orient to their name. By the age of 12 months, the child should be pointing to get things that they want. Lack of responding when name is called and lack of pointing are red flags that should be carefully evaluated. Goldson, E., & Reynolds, A. (2016). Child Development & Behavior. In W. W. Hay, M. J. Levin, R. R. Deterding, & M. J. Abzug (Eds.), Current diagnosis & treatment: Pediatrics (23rd ed., pp. 71-95). McGraw Hill Education. 1 54. A 33-year-old African American female at 35 weeks’ pregnancy presents to the ED complaining of headache, blurry vision, and sudden RUQ (right upper quadrant) pain for the last 2 days. She noticed her face, fingers, and ankles have swelling for the last 24hr. Physical examination is notable jugular venous distention and swelling of the legs. Her blood pressure is 170/110mmHg. Serum transaminase: 2x the upper limit of normal. Serum creatinine: 1.5 mg/dL. Urinalysis: 3+ protein. What is the most likely diagnosis? A. Placental abruption B. Myasthenia gravis C. Miscarriage D. Preeclampsia 55. A 2-year-old presents to the clinic for her 24 month well child visit. Which statement by the mother is concerning and warrants further investigation? A. “She in unable to hope on 1 foot.” B. “When things don’t go her way, she throws herself on the floor screaming, crying, and kicking.” C. “She doesn’t point at things to get her needs met, she is very independent and does things for herself. She even ignores me when I call her name.” D. “She doesn’t interact well when she plays with her 4-year-old brother. She usually plays alongside him instead of with him.” 56. A 45yo male patient presents to the clinic with complaints of decreased libido, erectile dysfunction, fatigue, and difficulty performing vigorous physical activity. Lab results indicate low serum testosterone. Which of the following statements would indicate the need for further evaluation before initiating testosterone therapy? A. “My wife says I snore a lot at night.” B. “I drink 2 cups of milk a day.” C. “Asthma runs in my family.” Page 61 of 114 1 D. “I take aspirin 81mg daily.” Page 62 of 114 Maintaining a normal body weight (BMI), 18.5-24.9) may result in a decrease of 5-20 mm Hg. The DASH diet consumes fruits, vegetables and low fat dairy products with reduced content of saturated fat and total fat 8-14 mm hg. Physical activity at least 30 min. A day 4-9 mm hg. Reduce dietary sodium intake to no more than 100 mEq/day 2-8 mm Hg. Papadakis, A. Maxine & McPhee, J. Stephen. (2019). Current Medical Diagnosis & HSV is diagnosed by cell culture, PCR or NAAT using bodily fluid or Serological Tests using blood. HSV is not detected in sputum, urine or stool. Reirden, D.H. & Nyquist, A. (2018). Sexual Transmitted Infections. In W.W. Hay Jr., R.R. Deterding, M.J. Levin, & M.J. Abzug (Eds.), Current diagnosis & treatment: Pediatrics (24th ed., pp. 1366). McGraw-Hill Education. STIs are considered a medical emancipated condition, therefore you do not need parental consent. Gonorrhea often has co-infection with chlamydia, so you will treat with both ceftriaxone and azithromycin, not individually. Reirden, D.H. & Nyquist, A. (2018). Sexual Transmitted Infections. In W.W. Hay Jr., R.R. Deterding, M.J. Levin, & M.J. Abzug (Eds.), Current diagnosis & treatment: Pediatrics (24th ed., pp. 1355-1360 ). McGraw-Hill Education. 1 62. In educating your patient about lifestyle modifications to manage hypertension, which is the most beneficial recommendation to lower blood pressure? A. Dash Diet B. Physical Activity C. Weight Reduction D. Decrease Sodium 63. A 17-year-old male presents to your clinic and states he has painful ulcerations to his penis that he noticed 1 week ago and began to feel tired and unwell about 2 weeks ago. Denies fever or dysuria or having these symptoms before. He admits to having unprotected sex with 2 partners recently. On examination, you notice vesicles to his penis and scrotum, with femoral adenopathy. You suspect HSV. What would you order to confirm? A. Blood sample B. Urine Culture C. Sputum sample D. Stool sample 64. A 14-year-old female comes to your clinic, unaccompanied by her parent with complaints of thick yellow discharge, pain on urination, and “my vagina feels like it's on fire”. States she noticed the symptoms 4 days ago and has had unprotected sex with her boyfriend. You perform a cervical exam and see a friable and edematous cervix. A urinalysis is negative, but a vaginal swab is being sent for a NAAT. Which one of the following would you tell your patient? A. I cannot treat you without your parent’s consent. Please make an appointment to return with a parent present B. I will treat you based on your symptoms with Ceftriaxone IM and azithromycin PO C. I will prescribe Ceftriaxone IM only D. I will prescribe Azithromycin PO only Page 65 of 114 Autism is characterized by 1) Abnormal development in social communication and interactions, and 2) Repetitive behaviors. This child is within the age range where screening for autism is important especially if parents present with concerns regarding social communication and interactions. The child’s abnormal speech development and repetitive behaviors of lining up toys and eats the same food every day is concerning for autism. B is Incorrect. Klinefelter syndrome is a disorder found only in males and usually at a later age when infertility is suspected. Even if the child were male, the case scenario does not demonstrate the signs and symptoms found in Klinefelter syndrome, such as breast enlargement and decreased body hair. C is Incorrect. Though the child’s abnormal speech development may be seen in a child with Down Syndrome, the other signs would prompt the practitioner to assess for autism. Furthermore, Down Syndrome can usually be diagnosed earlier than 18-months old due to current advancement in technology and early prenatal screening. D is incorrect. ADHD is usually diagnosed at a later age. Signs and symptoms of ADHD may be present prior to the age of 12. However, this case scenario demonstrates that a stronger differential diagnosis would be autism due to the patient’s age, lack of speech development, and repetitive behaviors. A is incorrect. Obesity is a risk factor for developing osteoarthritis. Therefore, weight loss can relieve the effects of osteoarthritis. This patient is obese with a BMI of 35, therefore, a diet promoting weight loss would be recommended. B is incorrect. Acetaminophen is a treatment for osteoarthritis, but the recommended daily dose should be no more than 2.6-4 g/day. Furthermore, acetaminophen is no longer first-line treatment for osteoarthritis. C is correct. Studies have shown that topical NSAIDs such as Diclofenac gel are more effective than placebos, particularly in treating knee osteoarthritis. Topical NSAIDs should be used early in the disease process. D is incorrect. Glucosamine did not show any increased effectiveness for treating osteoarthritis in comparison to placebo. 1 65. An 18-month-old girl is being seen today for a well-child visit. Upon interviewing the mother, she reports that her child does not speak much; says only “Mama.” She is also wondering if it is concerning that her child constantly lines up her toys, eats the same food every day, and shows no interest in playing with other children. What should you screen for? A. Autism B. Klinefelter syndrome C. Down syndrome D. Attention-Deficit/Hyperactivity Disorder 66. A 75-year-old female with a BMI of 35 presents to your clinic with reports of pain in bilateral knees. The pain starts in the morning but is brief and lasts for 15 minutes. The pain worsens with prolonged standing and improves with rest. Physical exam reveals Heberden nodes in both hands. Based on your primary diagnosis, what treatment is recommended? A. Diet high in carbohydrates to increase weight B. Acetaminophen 5 g/day PO as needed C. Topical NSAID - Diclofenac gel 1% applied to both knees QID D. Glucosamine for pain 67. When performing an assessment for an infant with suspected respiratory distress, an NP would expect to see all of the following clinical features EXCEPT: A. Intercostal & sternal retractions B. Expiratory grunting Page 66 of 114 1 C. Respiratory rate of 26 breaths per minute D. Cyanosis in room air Page 67 of 114 1 C. Fetal Alcohol D. Fragile X Syndrome Attention-Deficit/Hyperactivity is associated with a triad of symptoms of impulsivity, inattention, and hyperactivity. This child is 2 years old and is too young to be diagnosed with ADHD since this typical behavior at this age. Autism spectrum disorder can often be recognized in the first 12-18 months of life. The most common early characteristics are a consistent failure to orient to one’s name, regard people directly use gestures and develop speech. The question does not note any facial abnormalities or growth deficiencies Page 70 of 114 COPD is usually the combination of bronchitis symptoms and emphysema symptoms. Bronchitis is defined as a chronic cough of more than 3 months in 2 years and results with excessive mucus production in the bronchioles. Emphysema is the abnormal enlargement of air spaces, resulting in destruction of alveolar walls. COPD patients typically present with a persistent sputum production, cough, and dyspnea. Papadakis, M. A., McPhee, S. J., & Rabow, M. W. (2019). Chapter 9: Pulmonary Disorders. Current medical diagnosis & treatment. New York: McGraw-Hill Education. Teenage girls who are sexually active have a 1 in 8 chance of developing PID vs women in their 20’s who have one- tenth the risk. Those who have multiple sex partners and do not use condoms are at higher risk. Therefore it is important to educate the teenage population of the s/sx and prevention of PID. Hay, W. W. et al. (2018). Chapter 44 Sexually Transmitted Infections. In M. Haemer, L. Primak, & N. Krebs. Current diagnosis & treatment pediatrics (24th ed, pp 1361). Lange McGraw Hill. 1 which are characteristics of Fetal Alcohol syndrome. Fragile X syndrome is not a common in girls but of the 30% who are affected, characteristics include ADHD, anxiety, and shyness. Males may have symptoms of autism such as poor eye contact, hand flapping and tactile defensiveness. 74. It is important to realize when diagnosing your patient that COPD is often a combination of symptoms of which two pulmonary diseases? A. Asthma and Pneumonia B. Asthma and Emphysema C. Bronchitis and Asthma D. Bronchitis and Emphysema 75. Pelvic inflammatory disease (PID) is the most common gynecological disorder necessitating hospitalization for female patients of reproductive age. What age group has the highest incidence of PID? A. 20-30 years of age B. 13-19 years of age C. 31-40 years of age D. 41-50 years of age Page 71 of 114 Hypokalemia is a common side effect of loop diuretics (i.e. Lasix). Signs and symptoms of mild to moderate hypokalemia include fatigue, muscle cramps and weakness. Primary prevention helps reduce the risk of disease. By initiating a cessation plan, the clinician is aiding in preventing further diseases caused from cigarette smoking such as cardiovascular disease, Strokes, COPD, and cancer. Cigarette Smoking is still one of the highest factors related to death and morbidities in the United States. It is essential that tobacco use be discussed and documented at every visit. Data reports patients are more likely to try cessation if their provider advises them to quit. It is important to set goals that ate achievable such as a quit date. Pigone, M., & Salazar, R. (2019). Disease Prevention and Health Promotion. In M.A. Papadakis & S. J. McPhee (Eds.), Permethrin 5% is the treatment of choice for scabies. It should be applied as a single overnight application and repeated in 7 days to patient and entire household. Hay, W., Levin, M., Deterding, R. and Abzug, M., n.d. Current Diagnosis & Treatment. 24th ed. New York: McGraw- Hill Education, pp. 412. 1 Unit 5 Questions 1. A 62-year-old female presents to your clinic with new onset of muscle weakness, muscle cramps, and increased fatigue. The patient states that she also has increased urinary output since she has been prescribed Lasix (furosemide) for a recent diagnosis of CHF. What is the best clinical diagnosis for this patient according to her medical history? A. Hyponatremia B. Hyperkalemia C. Hypocalcem ia D. Hypokalemia 2. Jan, a 52-year-old female smoker, presents to your clinic for her annual wellness exam. As the practitioner, you discuss the risks associated with tobacco use. Jan states she has considering quitting, however she needs some form of nicotine cessation. After discussing several options with Jan, she agrees to try the Nicotine patch. Which type of health promotion and disease prevention is this an example of? A. Secondary prevention B. Primary prevention C. Tertiary prevention D. Smoking Cessation 3. What is the treatment of choice for scabies? A. Bactrim DS 800/160 BID x 5 days B. Permethrin 5% topical applied overnight and then repeat in 7 days. C. Fluconazole 400mg one-time dose. D. Warm compresses Page 72 of 114 Pyloric Stenosis is characterized by postprandial projectile vomiting. A palpable oval mass (olive-shaped) 5-15mm can be palpated in the right upper abdomen in 13.6% of cases, the treatment of choice is a Ramstedt pyloromyotomy. GERD is associated with postprandial regurgitation. The most common symptoms of duodenal ulcers in children less than 6 years is vomiting and upper GI bleeding. Hiatal Hernia in children > 48 months are associated with heartburn and regurgitation. Hoffenberg, E., Furuta, G., Kobak, G., Walker, T., Soden, J., Kramer. R., & Brumbaugh, D. (2018). Gastrointestinal Drugs account for over 70% of cases of acute interstitial nephritis. The most common drugs are penicillin medicines, cephalosporins, NSAIDs, PPIs and allopurinol. The classic triad of symptoms are fever, rash and arthralgias. Dirkx, Tonya C., Woodell,Tyler (2020). Kidney Disease. In Papadakis, M. A., McPhee, S. J. and Rabow, M.W. Current Medical Diagnosis & Treatment (59th ed., pp. 939-940). McGraw Hill Education. Acute glomerulonephritis causes hypertension and edematous with abnormal urinary sediment. Edema first presents in body parts with low tissue tensions such as scrotal and preorbital areas. Serum creatinine increases over days to months. BUN to creatinine ratio is not a dependable test of kidney function but rather volume status. Red cell casts are specific for glomerulonephritis. Dirkx, Tonya C., Woodell, Tyler (2020). Kidney Disease. In Papadakis, M. A., McPhee, S. J. and Rabow, M.W. Current 1 A. Hiatal Hernia B. Pyloric Stenosis C. GERD D. Duodenal Ulcer 8. A 55-year-old male presents to the clinic with fever, maculopapular rash and joint pain. Urinalysis shows white cells, red cells and white cell casts. Patient has been taking allopurinol for gout for the past 3 years. What is the best diagnosis for this patient? A. Glomerulonephritis B. Renal calculi C. Interstitial nephritis D. Urinary tract infection 9. A 65-year-old male presents to the clinic with hypertension, bilateral swelling of eyes and in the scrotal area. Urinalysis is obtained in clinic and reveals hematuria, mild proteinuria, and red cell casts. What is the best diagnosis with these presenting symptoms to clinic? A. Urinary tract infection B. Polycystic kidney disease C. Epididymitis D. Glomerulonephritis 10. A 3-year-old child presents to your clinic with a history of bilateral AOM. She was treated with Amoxicillin within the past 28 days. The mother states the child is not sleeping at night and is tugging at his right ear. On exam you note there is decreased motility with pneumatic otoscopy to the right tympanic membrane. There is pus behind Page 75 of 114 1 the TM, and it is reddened and retracted. The left TM is pearly white with a good cone of light. Boney landmarks visible. How would you treat today? Page 76 of 114 A bulging TM and middle ear effusion by pneumatic otoscopy or tympanometry must both be present to diagnose. If patient has taken amoxicillin in the past 30 days use Augmentin ES due to the assumed presence of the beta lactase. Augmentin is PCN with a beta lactam ring. Hay, W. W. et al. (2018). Chapter 18: Ear, Nose, & Throat. In Yoon, P.J., Scholes, M.A., & Friedman, N.R. (Eds.), Current diagnosis & treatment pediatrics Otitis Externa presents as edema and erythema of external canal which may be swollen shut, and severe ear pain made worse by movement of the pinna or tragus. Hay, W. W. et al. (2018). Chapter 18: Ear, Nose, & Throat. In Yoon, P.J., Scholes, M.A., & Friedman, N.R. (Eds.), Current diagnosis & treatment pediatrics (24th ed., pp. 478-479). Lange McGraw Hill. Initial treatment of nasolacrimal obstruction includes massaging lacrimal sac 6 times a day. Oral antibiotics (Augmentin) used to treat mild infection. The child should be referred to ophthalmology if persists beyond age 8 months or there is inflammation of lacrimal sac. IV antibiotics are used to treat severe infections or orbital cellulitis. Hay, W. W. et al. (2018). Chapter 16: Eye. In Jung, J.L. & Braverman, R.S.(Eds.), Current diagnosis & treatment 1 A. Amoxicilli n B. Augmentin C. Rocephin D. Omnicef 12. A high school swimmer comes into your urgent care. On exam you note, the right external canal is swollen and red. When you pull on the tragus the patient jumps and states “that was very painful”. You are unable to visualize the right TM due to the external canal being swollen shut. What is your presumptive diagnosis? A. Acute Otitis Media B. Furunculosis of the ear canal C. Otitis Externa D. Contact dermatitis of the ear canal 13. A mother brings her 1-month old infant into your clinic for a sick visit. The mother reports continuous drainage/tearing from the right eye. On exam you note yellowish discharge and mild swelling without erythema. How will you instruct the mother to treat at home? A. Have mother massage lacrimal sac 6 times a day. B. Augmentin C. Refer to Ophthalmology D. Admit for IV antibiotic 14. You are a nurse practitioner in the outpatient setting. You see a middle-aged businessman who complains of intermittent headaches on and off for the past couple of months. He states the headaches last about 30 minutes, are usually behind his L eye and sometimes the headaches wake him up in the middle of the night. You would classify Page 77 of 114 1 A. Rhinosinusiti s B. Allergic Rhinitis C. Vasomotor Rhinitis Page 80 of 114 Pale, boggy terminates are a predominate clinical feature of allergic rhinitis. Erythematous terminates may be present in rhinosinusitis, vasomotor rhinitis, and rhinitis Medicamentosa. Allergic rhinitis is usually accompanied with other features such as conjunctival erythema, teary eyes, and itching. Lustig, L. R., Schindler, J.S. Ear, Nose, & Throat Disorders. In Papadakis & McPhee, (2020). Current Medical Diagnosis & Treatment. McGraw Hill Lange. (ISBN: 978-1-260-45528-1) Intranasal corticosteroids are more effective and less expensive than antihistamines. Antihistamines can be used for temporary but immediate control. Antileukotrienes are adjunct medications. Intranasal anticholinergic agents work better for vasomotor rhinitis than allergic. Lustig, L. R., Schindler, J.S. Ear, Nose, & Throat Disorders. In Papadakis & McPhee, (2020). Current Medical Diagnosis & Treatment. McGraw Hill Lange. (ISBN: 978-1-260-45528-1) The patient presents with all four of the Centor criteria for group A beta-hemolytic streptococcal (GABHS) infection, which strongly suggests GABHS infection. Namenda (Memantine) is ineffective in frontotemporal dementia and may worsen cognition. Papadakis, M. A., McPhee, S. J., & Rabow, M. W. (2019). Chapter 24: Nervous System Disorders. In Current Medical Diagnosis & Treatment (58th ed., p. 1034-1035). New York, NY: McGraw-Hill Education. 1 D. Rhinitis Medicamentosa 18. What is the first line medication treatment for allergic rhinitis? A. Antihistamines B. Antileukotrienes C. Intranasal Corticosteroids D. Intranasal anticholinergic agents 19. An 18-year-old patient presents to your clinic with a chief complaint of sore throat. He denies cough. He has a fever of 101-degree F, tender swollen lymph nodes in the neck, and exudate is present on his tonsils. Which test is the most important to obtain? A. CBC B. UA C. Flu Swab D. Rapid Strep Test 20. Sally has recently been diagnosed with frontotemporal dementia. Which of the following should not be included in the education provided by the nurse practitioner? A. Namenda (Memantine) is a drug that will not cure this disease; however, it may slow down progression. B. The inability to make fine, precise movements of the right arm and leg is common. C. The patient may experience changes in emotions and social interactions D. It is important that the patient discontinues driving. Page 81 of 114 Hypercalcemia is a common sign of primary hyperparathyroidism. Hyperparathyroidism is usually affiliated with chronic symptoms, such as renal stones, and also less obvious symptoms such as fatigue, constipation and even frequent urination. Women between 50-60 years old are three times more likely to develop hyperparathyroidism. Papadakis, M. A., McPhee, S. J., & Rabow, M. W. (2019). Current medical diagnosis & treatment 2019, 905-908. New York: McGraw- Hill Education. Drugs account for over 70% of cases of interstitial nephritis, of which NSAIDS is on the list. Clinical features can include fever (more than 80% of cases), rash (25–50%), arthralgias, and peripheral blood eosinophilia (80%). The urine often contains white cells (95%), red cells, and white cell casts. Proteinuria can be a feature, particularly in NSAID-induced interstitial nephritis. Hays Chapter 24: Kidney and Urinary Tract Newborns and infants with UTI have nonspecific signs, including fever, hypothermia, jaundice, poor feeding, irritability, vomiting, failure to thrive, and sepsis. Strong, foul-smelling or cloudy urine may be noted. Because congenital urologic abnormalities increase the risk of UTI, a renal ultrasound, which is a noninvasive study, is recommended for children with UTI. The finding of significant hydronephrosis or other concerning urinary tract abnormalities on screening ultrasound warrants further imaging. 1 21. A 58-year-old female patient presents complaining of being easily fatigued, frequent urination, and constipated. This patient has a history of kidney stones for over 10 years. What electrolyte would you want to check? A. Potassiu m B. Calcium C. Sodium D. Magnesium 22. A 61-year-old female patient presented in your clinic complaining of a new rash, fever, and blood in her urine that started 2 days ago. The patient stated that in the past few weeks, she had taken 600mg ibuprofen three times a day for joint pain. A urine dipstick is positive for pyuria, hematuria and proteinuria. Which of the following is the patient most likely to experience? A. Urinary tract infection B. Glomerulonephritis C. Nephrolithiasis D. Interstitial nephritis 23. A mother brought in her 12-month-old child for fever, irritability, vomiting, and a foul-smelling odor from the infant’s diaper that started one day ago. Urine screening is positive for white blood cells in the urine. You suspect that this child has a congenital urological abnormality due to having a UTI previously. Which of the following imaging studies would you order first for this patient? A. Voiding cystourethrogram (VCUG) B. Renal ultrasound C. Renal scan D. Plain abdominal films 24. A 45-year-old with a history of chronic kidney disease, is complaining of chronic back Page 82 of 114 1 28. When performing a pediatric well-child exam of a newborn, it is crucial to check during the ophthalmic examination. Page 85 of 114 A. Snellen charts are not to be used until children are 3 years old. B. CORRECT- a red reflex that is abnormal may be attributed to life-threatening conditions (retinoblastoma) or vision problems (congenital cataracts). Additionally, cloudy corneas can cause an abnormal red reflex which could indicate congenital glaucoma, among others. C. Blood pressure is not assessed until age 3 at well-child visits and is not a crucial part of the ophthalmic exam. D. Audiometry using conventional screening, such as having the child raise a hand when they hear sound is not recommended until age 4. Also, the whisper test is not a crucial part of the newborn ophthalmic examination. Taking a bottle to bed is a poor dietary habit, placing the child at higher risk for dental caries. Oral hygiene needs to begin soon after birth, using a soft washcloth to cleanse the gums. After eruption of teeth, it is very important to practice in earnest. Most cases of anterior epistaxis may be treated by direct pressure of the nares continuously for 15 minutes. Venous pressure is reduced when slightly leaning forward and in the sitting position to lessen the swallowing of blood. Short-acting topical nasal decongestants, which act as vasoconstrictors, may also help. Avoiding nasal trauma such as nose picking, and blowing is important in stopping and preventing nose bleeding. Reference: Papadakis, M. A., McPhee, S. J., & Rabow, M. W. (2019). Chapter 8: Ear, Nose & Throat Disorders. In Current medical diagnosis & treatment, 2019 (pp. 231-232). New York: McGraw-Hill Education 1 A. visual acuity with a Snellen chart B. the red reflex C. blood pressure D. the whisper test 29. Which of the following is a TRUE statement regarding pediatric dentistry? A. Taking a bottle to bed is not a risk factor for childhood dental caries. B. Oral hygiene is not necessary until age 8, when a child can perform this function independently. C. Between 12-18 months old, infants should be weaned from the bottle and encouraged to use a cup with no cover to aid in prevention of dental caries. D. Applying fluoride varnish during well-child visits is not recommended to decrease dental caries in vulnerable children. 30. A 43-year-old male comes into the clinic for a wellness exam. During the visit he tells you he has been suffering from nose bleeds with the dry weather lately and asks your advice on how to stop a nosebleed quickly and sufficiently. What answer is best? A. Lean your head back while pinching the soft part of the nares below the nasal bone B. Apply continuous direct pressure to the nose for 5 minutes C. Use a short acting topical nasal decongestant D. Blow your nose to help stop the bleeding 31. A mother brings her 8 y/o daughter into the clinic stating she has been c/o sore throat worse with swallowing and running a fever of 101⁰F. On exam the patient has palatal petechiae, bilateral cervical lymphadenopathy and a positive rapid strep test. The patient has a PCN allergy. What medication would you prescribe to treat the patient’s Page 86 of 114 1 streptococcal pharyngitis? Page 87 of 114 Children respond to decreased circulating volume by compensatory tachycardia but may have a normal blood pressure even with severe dehydration. Low or falling blood pressure in a child is a late sign of shock and requires emergent treatment. Hanna, M. G., & Bock, M. (2018). Fluid, electrolyte, & acid-base disorders & therapy. In W. W. Hay, R. R. Deterding, In calculating pediatric maintenance fluid replacement, first convert the weight to kg. Then calculate 100 mL of fluid/kg/24hr for the first 10kg body weight. Next calculate 50mL of fluid/kg/24hr for the second 10kg of body weight. Then calculate 10 to 25mL of fluid/kg/24hr for every kg over 20kg of body weight. Finally, add the 3 volumes together to determine how much fluid should be replaced in 24 hours. Calculation: 55 lb = 25 kg 100 mL x 10 kg = 1000 mL (a) 50 mL x 10 kg = 500 mL (b) Range of 10 to 25 mL x 5 kg = 50 mL to 125 mL (c) Add (a) + (b) + (c) => 1000 mL + 500 mL + 10mL = 1550 mL UP TO 1000 mL + 500 mL + 125 mL = 1625 mL (Side Note: Recall that when forced fluids are required based on patient condition, the minimum replacement is at least 1.5 times the maintenance amount.) Somers, M. J. (2019, June 19). Maintenance intravenous fluid therapy in children. UpToDate. https://www.uptodate.com/contents/maintenance-intravenous-fluid- 1 Q2) According to Current Pediatrics immunization for immunodeficient children the contraindication to live vaccines does not apply if the child had cancer, is in remission, and has not had chemotherapy in at least 90 days. So vaccination can resume on a catch-up schedule once these criteria are met. (Daley et al., 2018, p. 254) 35. In the pediatric clinic, the FNP is evaluating a 5-year-old patient complaining of nausea, vomiting, & diarrhea for 2 days. The physical exam reveals dry mucous membranes, increased capillary refill time, tachycardia, and hypotension. What is the FNPs most appropriate next action? A. Order CBC & CMP to assess for anemia or electrolyte imbalance B. Send the child to the ER as these signs indicate a medical emergency C. Push oral fluids in the office and reassess in 15 minutes D. Watchfully wait while parents increase oral fluid intake at home; reassess in 24 hours 36. A pediatric patient weighing 55 pounds is being treated for dehydration. How much fluid should be replaced in the first 24 hours for this patient? A. 3050 mL B. 2200 mL C. 1575 mL D. 825 mL 37. Due to the danger of causing discoloration to the emerging permanent teeth under the skin, which antibiotic should be avoided in children below the age of 12? A. Amoxicillin B. Keflex Page 90 of 114 1 C. Cipro D. Doxycycline Page 91 of 114 The NP would ask what formula the NB is being feed. Failure to thrive, lethargy, and fever are all symptoms of excessive protein intake in preterm infants that have had more than 6g/kg/day over a 1-2-week period. Hay, W. W., Levin, M. J., Deterding, R. R., & Abzug, M. J. (2018). Oral medicine & dentistry. In u. Klein, R. P. Gandhi, & C. V. Gannam (eds.), Current diagnosis & treatment pediatrics (24th ed., pp. 281-282). McGraw Hill Education. 2) The patient’s symptoms are all signs of mastitis. When treating mastitis, the antibiotic is to be given for 10 days which makes C the only correct answer. Hay, W. W., Levin, M. J., Deterding, R. R., & Abzug, M. J. (2018). Oral medicine & dentistry. In u. Klein, R. P. Gandhi, & C. V. Gannam (eds.), Current diagnosis & treatment pediatrics (24th ed., pp. 292-293). McGraw Hill Education. 1 According to Hay, Tetracyclines pose a risk, to children below the age of 12, of discoloring the permanent teeth that are developing. For this reason, tetracyclines are avoided, and Amoxicillin or Penicillin are often the key choice as a substitute to tetracyclines in the setting of dental traumatic injuries. Hay, W. W. et al. (2018). Current diagnosis & treatment pediatrics (24th ed.; pp. 472, 475). Lange McGraw Hill. 38. A mother comes into the clinic with a month-old newborn born at 35 weeks’ gestation weighing 2.7 kg that was discharged from the NICU 3 week ago. The mother tells you that a week ago her baby was drinking a formula bottle every 2 hours and doing great. She tells you for the last week her baby is not wanting to eat, temperature 100.9 F, drowsy when awake, and sleeping longer than usual. What question would the NP anticipate asking during the assessment? A. What formula are you feeding your NB? B. Have you been to the pediatrician? C. How many wet diapers in a 24-hour period? D. How many bowel movements in a 24-hour period? 39. A new mother is seen in the office today with flulike symptoms. When entering the room, you notice that the pt is breast feeding. The following objective data is found on assessment: left breast tenderness, erythema, and firmness. What antibiotic would the NP prescribe for this patient? A. Cephalexin x7 days B. Dicloxacillin x10 days C. Doxycycline x 8 days D. Bactrim DS x 5 days 40. A child is brought to the clinic by his parents due to itchy, watery, red eyes. He also has sneezing and a runny nose. The nurse practitioner knows that this ophthalmologic issue can be safely treated with any of the following except: A. An ophthalmic nonsteroidal anti-inflammatory B. Avoidance of allergens C. An ophthalmologic mast cell stabilizer D. Long-term ophthalmologic corticosteroids Page 92 of 114 Metformin should be discontinued when the eGFR is less than 30 ml/min because of the risk of lactic acidosis. Lactic acidosis if left untreated can be life threatening. M. Papdakis & S. McPhee (2019). Current Medical Diagnosis and Treatment (58th Ed, pp. 941). Mcgraw-Hill Education. The patient is experiencing an allergic reaction but there is no signs or symptoms that reflect immediate anaphylaxis. Therefore, treating the patient with PO antihistamines and PO corticosteroids would be an appropriate option. The NP should prescribe an EpiPen in the event exposure to the antigen occurs with a more serious reaction. Diphenhydramine should not be given in clinic do to risk of dizziness/drowsiness and risk for injury operating machinery. While prescribing a topical steroid will not hurt, it is not the best option for treating this patient. Shinkai, K., & Fox, L. (2019). Dermatologic Disorders. In M. A. Papadakis & S. J. The initial lesion often called the “herald patch” is usually significantly larger than the other lesions and often occurs 1-2 weeks prior to the lesions that erupt later. None of the other answers are correct. Prok, L., & Torres, C. (2018). Skin. In W. W. Hay, M. J. Levin, R. R. Deterding, & M. J. Abzug (Eds). Current diagnosis & treatment pediatrics (24th ed., pp. 416). McGraw-Hill Education. 1 A. Liraglutide B. Simvastat in C. Metformin D. Losartan 44. A 25-year-old patient presents to the clinic with pruritic hives on her face, arms, trunk, and legs. She has no shortness of breath, wheezing, difficulty swallowing, or swelling of the lips or tongue. With the provided information, which option would be appropriate for the NP to provide for this patient? A. A dose of Epinephrine should be administered STAT via EpiPen B. The patient should be prescribed a PO antihistamine and PO corticosteroid C. The patient should be prescribed a topical corticosteroid x1 week or until lesions resolve due to having no systemic symptoms D. Administer a one-time dose of Diphenhydramine in clinic STAT 45. A 16-year-old male patient reports an ongoing rash for the last two weeks. He reports having a slightly raised, scaly patch to his side that he noticed about one week prior to the rash appearing. During the exam you note several small, pink, oval, lesions scattered on his trunk. He denies pruritis or pain associated with the lesions. There is one lesion considerably larger observed on side of his abdomen. The APRN diagnosis the patient with which condition? A. Rosacea B. Pityriasis rosea C. Lichen planus D. Eczema Page 95 of 114 1 46. A 58-year-old woman arrives at the clinic for lab work. The patient has a history of chronic kidney disease stage 1, hypertension, and hyperlipidemia. The patient is currently taking pravastatin, lisinopril, Page 96 of 114 Pravastatin is not nephrotoxic and can be taken in patients with a history of chronic kidney disease. Lisinopril can cause hyperglycemia. Naproxen can be nephrotoxic, and the patient has a history of CKD stage 1. Therefore, Lisinopril and Naproxen both need to be adjusted for the patient’s safety. Hypocalcemia causes muscle spasms, seizures, and arrhythmias. Hypermagnesemia causes hypotension, bradycardia, and cardiac arrest. Hyperkalemia causes abdominal cramping, fatigue, muscle weakness, paralysis, and cardiac arrest. Hypernatremia causes neurological changes, muscular twitching, hypertension, and weakness. Hanna, M. G., & Bock, M. (2018). Chapter 23: Fluid, Electrolyte, and Acid-base Disorders & Therapy. In W. W. Hay, M. J. Levin, R. R. Deterding, & M. J. Abzug (Eds), Current diagnosis & treatment. New York, New York: McGraw-Hill Renal ultrasound would be the most definitive test out of the four as well as least costly. A chest x-ray would not show any kidney stones. A urinalysis can have RBC’s present but does not definitively confirm presence of a kidney stone, and an MRI of the abdomen can be costly when an ultrasound could suffice. 1 hydrochlorothiazide(HCTZ), and naproxen PRN for joint pain. Lab results show that the patient’s potassium is 5.4. Which medications should be adjusted for the safety of the patient? A. Pravastatin B. Lisinopril C. Naproxe n D. Both B&C 47. BP readings at home, muscle twitching, decreased alertness, and overall weakness. Lab results diagnose the patient with acute kidney injury. Which electrolyte imbalance do you suspect is causing the patient’s symptoms? A. Hypernatremia B. Hyperkalemia C. Hypermagnesemia D. Hypocalcemia 48. A 65-year-old female patient arrives at the office complaining of some intermittent nausea & vomiting. Pt also reports some flank pain and difficulty urinating. Which test can you order to confirm the presence of kidney stones? A. Chest X-ray B. Renal Ultrasound C. Urinalysis D. MRI of abdomen 49. A 70-year-old man with a history of chronic kidney disease is coming in to review lab results in the office today. The patient's eGFR is 55 which diagnoses the patient with chronic kidney disease stage 3. Which types of medications would you be advising the patient to avoid? A. Acetaminophen Page 97 of 114
Docsity logo



Copyright © 2024 Ladybird Srl - Via Leonardo da Vinci 16, 10126, Torino, Italy - VAT 10816460017 - All rights reserved